MSW Practice Test_ MSW Advanced Standing-2022 Fall Quarter 08_29-11_20 (1)

pdf

School

Long Beach City College *

*We aren’t endorsed by this school

Course

1

Subject

Sociology

Date

Jan 9, 2024

Type

pdf

Pages

65

Uploaded by claytonselisa18

Report
11/3/23, 10:36 PM MSW Practice Test: MSW Advanced Standing-2022 Fall Quarter 08/29-11/20 https://waldenu.instructure.com/courses/17315/quizzes/13999 1/65 Due No due date Points 170 Questions 170 Time Limit 240 Minutes Allowed Attempts Unlimited Instructions Attempt History Attempt Time Score LATEST Attempt 1 240 minutes 21 out of 170 Score for this attempt: 21 out of 170 Submitted Nov 3 at 9:13pm This attempt took 240 minutes. MSW PRACTICE TEST Ready to try it out? In this, section, you have the opportunity to take a practice version of the ASWB exam. As with the real exam, you will have 4 hours to complete this objective, 170-question assessment. For each item, you will receive feedback on the answer. Take the test as many times as you would like. Practice makes perfect! The ASWB exam includes 170 total questions. Of those questions, 150 are scored. In general, a passing score ranges from 90 – 107 depending on difficulty level of the questions. In this practice exam, you will need to answer 110 questions correctly to attain a passing score. Please review the ASWB Exam Scoring web page (https://www.aswb.org/exam/exam-scoring/) for more information on scoring the ASWB Masters Exam. Take the Quiz Again 0 / 1 pts Question 1 To conform to the principles of informed consent, a social worker must obtain written permission from the client to release information before: consulting with the social worker's supervisor about the client You Answered You Answered discussing the client's situation with a member of the client's immediate family Correct Answer Correct Answer coding client identifying information in an agency's computerized data system recording identifying information in a client record The only option among those listed that requires written consent from the client is when discussion takes place with the client’s family members. The social worker may want to make the client aware of the situations mentioned in options (A), (C), and (D), but written permission from the client is not required. 1 / 1 pts Question 2 A 46 -year-old female client complains that for the past six months, she has lost pleasure in activities she had previously enjoyed. She is also having difficulty getting up in the morning and finds she is more irritable than usual. Her youngest child has recently left home and her husband is continuing to work long hours. The MOST appropriate focus for the initial intervention is: interviewing the husband and children
11/3/23, 10:36 PM MSW Practice Test: MSW Advanced Standing-2022 Fall Quarter 08/29-11/20 https://waldenu.instructure.com/courses/17315/quizzes/13999 2/65 referring the client to a support group exploring the client's losses Correct! Correct! referring the client for medication evaluation The factors mentioned by the client represent losses that she has experienced and, as such, must be explored (key C). Options (A), (B), and (D) are actions the social worker may take once the client’s situation is understood. 1 / 1 pts Question 3 Which of the following characteristics BEST describes normal development in a two-year-old child? Developing friendships Being toilet trained Counting to 20 Being determined to do for one's self Correct! Correct! According to psychosocial models of development, two-year-olds are going through a stage of establishing independence from parents, wanting to make decisions for themselves (key D). They are not yet developing friendships, option (A). Options (B) and (C) are things that are taught, rather than being considered developmental. 0 / 1 pts Question 4 A social worker is concerned that an article in a professional periodical contains elements of ethnic bias. How can the social worker MOST effectively address this concern? Write a letter to the journal editors Correct Answer Correct Answer Cancel the subscription to the journal Report the author to the licensing board You Answered You Answered Have coworkers sign a resolution objecting to the article In order to encourage change that would address the social worker’s ethical concerns, the social worker must inform the editors of the problem (key A). Forwarding concerns to the editors would allow corrections or retractions to be made. Canceling the subscription, option (B), would not convey the social worker’s concern. Reporting the author to the board, option (C), would not be appropriate at this point. Having coworkers sign a resolution, option (D), might be a possibility if the problem continues or if the initial letter goes unanswered, but it would not be the MOST effective or direct way to address the concern.
11/3/23, 10:36 PM MSW Practice Test: MSW Advanced Standing-2022 Fall Quarter 08/29-11/20 https://waldenu.instructure.com/courses/17315/quizzes/13999 3/65 0 / 1 pts Question 5 Which of the following concepts is concerned with whether an instrument is measuring what it purports to measure? Statistical regression Statistical significance You Answered You Answered Validity Correct Answer Correct Answer Reliability This is another recall question, requiring knowledge of the definition of validity in testing. 1 / 1 pts Question 6 A social worker at a mental health center meets with a client for an intake session. The client reports hearing voices warning that “people are out to get me.” The social worker notices persistent incoherent speech and poor personal hygiene coupled with inappropriate clothing for the current season. Based on the described symptoms and behaviors, which disorder should the social worker suspect? Bipolar I Somatic Schizophrenia Correct! Correct! Anxiety The symptoms of hearing paranoid voices, having incoherent speech, poor hygiene and inappropriate clothing are consistent with the criteria for schizophrenia (key C). There is no information in the stem that would support the likelihood of (A), (B), or (D). 0 / 1 pts Question 7 A social worker has direct knowledge that a colleague has been drinking excessively, missing appointments, and showing up late for work. The social worker is concerned that clients may be negatively affected by this behavior. The social worker meets with the colleague and presents concerns, but no changes are made by the colleague. The social worker should NEXT: hold a joint meeting with the supervisor and the colleague to discuss the problem Correct Answer Correct Answer ask the supervisor to reassign the colleague's clients to another social worker You Answered You Answered immediately report the colleague to the jurisdictional regulatory body
Your preview ends here
Eager to read complete document? Join bartleby learn and gain access to the full version
  • Access to all documents
  • Unlimited textbook solutions
  • 24/7 expert homework help
11/3/23, 10:36 PM MSW Practice Test: MSW Advanced Standing-2022 Fall Quarter 08/29-11/20 https://waldenu.instructure.com/courses/17315/quizzes/13999 4/65 report the colleague's behavior to the executive director of the agency Following social work ethical protocol, the NEXT step would be to discuss the concerns with the supervisor, and the colleague may be included (key A). At that time, it would be determined if the behavior would need to be reported to the executive director. If the colleague’s behaviors continue, then it becomes appropriate to report to the regulatory body. Option (B) enables the social worker to continue the inappropriate behaviors and does not address the issue or protect the clients. 0 / 1 pts Question 8 After the assessment stage, how can the social worker successfully obtain the client’s commitment to social skills training? Model the selected social skill several times Role play specific social skill components You Answered You Answered Discuss the rationale for social skill development Correct Answer Correct Answer Administer a social skills pretest Options (A) and (B) may be helpful in teaching social skills, but until the client understands the rationale for learning the skills, it will be difficult to commit to the process. Option (D) would do nothing to enhance commitment. 1 / 1 pts Question 9 A chronically mentally ill client appears at a homeless shelter. The client lives on the street and wants to return home. Maslow's hierarchy of needs would place this client at what level? Self-esteem Safety Correct! Correct! Physiological Belonging This item makes use of Maslow’s hierarchy of needs. The basic need for safety or shelter must be met before a person can successfully meet higher level needs. 1 / 1 pts Question 10
11/3/23, 10:36 PM MSW Practice Test: MSW Advanced Standing-2022 Fall Quarter 08/29-11/20 https://waldenu.instructure.com/courses/17315/quizzes/13999 5/65 A hospitalized client has been declared incompetent and a friend has been appointed by the court to act for him. The social worker should: carry out the wishes of the legal representative consult with the physician and carry out the physician's orders meet with the legal representative and the physician to discuss the case Correct! Correct! see that the client is discharged as soon as possible In this instance, both the guardian and the patient’s physician should be involved in any planning that is done on behalf of the patient. Neither should make decisions without input from the other. There is no indication that discharge is being considered at this time. Families (key D). 1 / 1 pts Question 11 A social worker at a substance abuse clinic is conducting an initial interview with a client. To evaluate the client’s ability to use services, the social worker should examine the client’s: available social supports acceptance of the problem Correct! Correct! ego strengths level of stress caused by the problem This is an unscored item. The client must first recognize that there is a problem before making effective use of the services offered by the agency. The remaining three options would be addressed as a part of the social worker’s intervention, but they will not determine the client’s ability to use services. 0 / 1 pts Question 12 A social worker's selection of clients to participate in a research project regarding child abuse is BEST described as: recruitment evaluation You Answered You Answered sampling Correct Answer Correct Answer assessment
11/3/23, 10:36 PM MSW Practice Test: MSW Advanced Standing-2022 Fall Quarter 08/29-11/20 https://waldenu.instructure.com/courses/17315/quizzes/13999 6/65 The process of identifying the part of a population for study participants is known as sampling (key C). The only other option that is part of the preliminary part of the project, recruiting, option (A), is actually inviting participants. 0 / 1 pts Question 13 During a session with a social worker, a client discloses that she is a lesbian. The social worker is morally against homosexuality. What is the appropriate action for the social worker to take? Discuss the value conflict openly with the client Ask the supervisor to transfer the case immediately You Answered You Answered Suspend personal values to continue to work with the client Correct Answer Correct Answer Refer the client to a social worker who is gay It would not be appropriate to discuss the social worker’s personal values with the client, option (A), nor would it be effective to refer the client to a social worker who is gay or transfer the case, options (B) and (D). It is a reasonable expectation for the social worker to suspend personal values in order to meet the client’s needs (key C) while the issues are worked out through supervision. 0 / 1 pts Question 14 A school social worker receives a referral on a student who has recently become withdrawn and whose grades have declined. During an interview, the social worker learns that one parent has just been incarcerated for domestic abuse. In preparing the social history for the school, the social worker has an obligation to: inform the school staff of all of the details of the incident review the police reports for collaboration of the event You Answered You Answered interview the neighbors for their perspective on the family provide only the information that affects the child's education Correct Answer Correct Answer The social worker is obligated to maintain client confidentiality to the greatest extent possible, (key D), by not giving information to anyone else when it is not necessary 0 / 1 pts Question 15 A social worker in a public health center is working with a client diagnosed with HIV. The social worker does not have a strong knowledge base in issues related to HIV and seeks information from another social worker who is an HIV educator. This contact is an example of:
Your preview ends here
Eager to read complete document? Join bartleby learn and gain access to the full version
  • Access to all documents
  • Unlimited textbook solutions
  • 24/7 expert homework help
11/3/23, 10:36 PM MSW Practice Test: MSW Advanced Standing-2022 Fall Quarter 08/29-11/20 https://waldenu.instructure.com/courses/17315/quizzes/13999 7/65 professional consultation Correct Answer Correct Answer interdisciplinary collaboration You Answered You Answered a multidisciplinary approach formal networking Obtaining information from a colleague is professional consultation. Since the social workers do not share the client, the contact would not be considered interdisciplinary collaboration. Because they are both social workers, they are from the same discipline, so it is not a multidisciplinary approach. This is not an example of formal networking, which is establishing links for the purpose of having a support system in place. 1 / 1 pts Question 16 A social worker has been involved in a long and extremely intense session with a client who is profoundly depressed about the recent death of his wife. Toward the end of this unsuccessful session, the client states, “I have a gun and I will use it when I get home.” The social worker must: continue to help the client deal with the recent loss of the spouse facilitate a hospitalization for the client Correct! Correct! contact the client’s family to discuss safety concerns contact adult protective services There is clear indication that the client intends to commit suicide. The client’s behavior and comments clearly show that he must be protected, and hospitalization will provide the safety he needs. It is not appropriate for adult protective services to be involved. 1 / 1 pts Question 17 A social worker makes this notation in a client's record: "Client speaks with a very heavy accent; difficult to understand." The entry reflects the social worker's: stereotyping multiculturalism ethnocentrism Correct! Correct! poor communication skills
11/3/23, 10:36 PM MSW Practice Test: MSW Advanced Standing-2022 Fall Quarter 08/29-11/20 https://waldenu.instructure.com/courses/17315/quizzes/13999 8/65 Ethnocentrism (key C) is the belief that one culture is superior to another, in this case a conviction that unaccented English is the appropriate method of communication. Option (A), stereotyping, refers to an overgeneralization that is not reflected here. Option (B) is not reflected here, and option (D) also has no basis in the stem, since the social worker might have excellent communication skills in other situations where he or she is better able to understand the client. 0 / 1 pts Question 18 A family member brings several concerns to a nursing facility social worker. To BEST understand the family member's issues, the social worker should use which interviewing technique? Paraphrasing Reflection Empathy You Answered You Answered Clarification Correct Answer Correct Answer Options (A), (B), and (C) are interviewing techniques that can facilitate communication with the client but do not necessarily lead to better understanding of the concerns. The use of clarification can best specify the issues of concern to the family member. 1 / 1 pts Question 19 A social worker meets with a client who is elderly and reports increasing problems with memory. What action would be MOST appropriate for the social worker to take? Assure the client that memory loss is part of the aging process Encourage the client to obtain a medical evaluation Correct! Correct! Refer the client and family to the Alzheimer's Association Monitor the client's memory loss over a two-month period The role of the social worker is to first make a referral to rule out medical causes (key B) for the decline in memory. Options (A) and (B) are actions the social worker might include with the client once physical causes have been ruled out. Option (D) would not be appropriate for the social worker to do, since it should be done within the scope of practice of a medical evaluation. 1 / 1 pts Question 20
11/3/23, 10:36 PM MSW Practice Test: MSW Advanced Standing-2022 Fall Quarter 08/29-11/20 https://waldenu.instructure.com/courses/17315/quizzes/13999 9/65 Parents of a recently married daughter report that they are uncertain about how to relate to the newlyweds. Which of the following concepts BEST describes the experience of the parents? Generational conflict Normal transition Correct! Correct! Dysfunctional adaptation Enmeshed relationship Initial anxiety surrounding role transition is normal behavior (key B). There is no information in the stem that would justify options (A), (C), or (D). 0 / 1 pts Question 21 Which of the following steps is an appropriate part of the referral process? Following up to determine whether the resources were provided Correct Answer Correct Answer Alerting the client to the unprofessional behavior he or she can expect from the agency to which he or she has been referred Offering the client the use of the worker's resource listing, but allowing him or her to determine whom to call You Answered You Answered Eliciting family history in order to provide the person to whom the client is referred with complete information (Key A) is the only option that is part of the process. Option (B) is inappropriate, (C) is empowerment that negates truly making a referral, and (D) would be done by the professional to whom a referral is made. 0 / 1 pts Question 22 A social worker has been assisting a neighborhood organization in working with a city housing department, urging it to demolish abandoned and unsafe homes in its area. The committee members have asked the social worker to help them develop an instrument that will aid them in determining their level of success in this project. The social worker presents them with an evaluation plan that lists outcomes from "much less than expected" to "much more than expected." This plan is what kind of evaluation tool? Goal-attainment scaling Correct Answer Correct Answer Consumer satisfaction survey You Answered You Answered Target-problem scaling Rapid assessment instrument
Your preview ends here
Eager to read complete document? Join bartleby learn and gain access to the full version
  • Access to all documents
  • Unlimited textbook solutions
  • 24/7 expert homework help
11/3/23, 10:36 PM MSW Practice Test: MSW Advanced Standing-2022 Fall Quarter 08/29-11/20 https://waldenu.instructure.com/courses/17315/quizzes/13999 10/65 (Key A) is the only option that deals with a group project. Target-problem scaling, option (C), would assist in identifying the problem, as would option (D). Option (B), like (key A), is an outcome measure, but the committee members are not consumers. 0 / 1 pts Question 23 In investigating a child abuse report, the social worker should FIRST: do a psychosocial assessment You Answered You Answered develop a treatment plan evaluate the danger to the child Correct Answer Correct Answer write an intake disposition When a social worker is responding to a report of abuse, safety is the FIRST concern (key C). The other options might be done later on, depending on the setting. 0 / 1 pts Question 24 What is the MOST effective way a social worker can communicate acceptance to a client? Agree with the client's perception of the problem Prioritize the client's presenting problem You Answered You Answered Educate the client on possible solutions for the problem Allow the client to fully explain the problem Correct Answer Correct Answer While options (A), (B), and (C) are positive responses to a client, they represent control by the social worker, not by the client. (Key D) suggests client control of the conversation, indicating that the social worker respects the information presented by the client. 1 / 1 pts Question 25 A social worker at a psychiatric facility is preparing for the initial team meeting on a recently admitted client. What should the social worker be prepared to do at this meeting? Present the client's psychosocial information Correct! Correct! Make referrals for needed services
11/3/23, 10:36 PM MSW Practice Test: MSW Advanced Standing-2022 Fall Quarter 08/29-11/20 https://waldenu.instructure.com/courses/17315/quizzes/13999 11/65 Discuss the results of the client's mental status exam Serve as moderator for the meeting The social worker’s expertise includes the client’s person-in-environment information (key A) that is shared with the team. This enables the other team members to view the client in a holistic manner and make appropriate referrals, if needed. Options (C) and (D) might be correct in some team circumstances; but in a psychiatric facility, these roles can be expected to be taken to a psychiatrist. 0 / 1 pts Question 26 In the process of assessing whether to use another agency’s records as collateral data, the social worker should: assume this information is authentic because of its source avoid using any information that does not come from the client rely only on information documented by professionals You Answered You Answered determine relevance of the information to the current situation Correct Answer Correct Answer For various reasons, the client may omit certain information when being interviewed by the social worker. Information obtained with the client’s consent from other sources in the community can be very helpful in completing an initial assessment. However, the information requested should be relevant to the client’s situation and experience. 1 / 1 pts Question 27 Which aspect of a client's behavior during an interview would BEST help the social worker to assess the communication skills of the client? Degree of openness with which the client discusses concerns Ability of the client to remain focused during the interview Correct! Correct! Number of times the client requests clarification Congruence between verbal and nonverbal behaviors of the client Only (key B) is about basic communication skills. Option (A) is emotional openness, while (C) is a cognitive process rather than a communication skill. (D) is about assessment of the client’s current emotional state, not communication. 0 / 1 pts Question 28
11/3/23, 10:36 PM MSW Practice Test: MSW Advanced Standing-2022 Fall Quarter 08/29-11/20 https://waldenu.instructure.com/courses/17315/quizzes/13999 12/65 When legislative changes decrease government support for social welfare programs, social workers should FIRST: organize communities to advocate for social change strengthen community networks and community practices You Answered You Answered closely monitor the impact of the changes on clients Correct Answer Correct Answer empower families to better care for themselves Although social workers would probably do options (A), (B), and (D) at some point, (key C) would be FIRST because it is the most immediate action to take. (D) in particular could not be done until the needed information to be found by monitoring the impact of the changes is available. 0 / 1 pts Question 29 An empathic response includes verbalizing the social worker's understanding and: mirroring the client's feelings Correct Answer Correct Answer expressing the social worker's similar feeling You Answered You Answered exploring the problem further universalizing the client's problem An empathic response communicates to the client that the social worker clearly understands his or her emotions (key A). Empathy is not the social worker actually experiencing the same feeling, option (B), or generalizing the problem, option (D). 0 / 1 pts Question 30 A 15 -year-old client confides to her social worker that she is two months pregnant and does not know who the father is. The social worker should FIRST: refer the client to a support group for teenage mothers advise the client to talk with her parents or guardian refer the client for immediate prenatal care You Answered You Answered help the client identify her worries and concerns Correct Answer Correct Answer
Your preview ends here
Eager to read complete document? Join bartleby learn and gain access to the full version
  • Access to all documents
  • Unlimited textbook solutions
  • 24/7 expert homework help
11/3/23, 10:36 PM MSW Practice Test: MSW Advanced Standing-2022 Fall Quarter 08/29-11/20 https://waldenu.instructure.com/courses/17315/quizzes/13999 13/65 This is a clear example of starting where the client is. The other alternatives may come later in the process of working with the girl. 0 / 1 pts Question 31 A social worker was asked to conduct a home visit with an elderly widow to assess living conditions. The social worker observes unlaundered clothing in the living room, pervasive pet odors, and spoiled food in the kitchen. The woman appears confused and disorganized. The social worker should FIRST: refer the client to homemaker services contact adult protective services You Answered You Answered determine family availability evaluate the client's needs Correct Answer Correct Answer In order to determine an action plan, the social worker needs to understand why the present conditions are occurring. Options (A), (B), and (C) may be appropriate later in the process. 0 / 1 pts Question 32 In an intake interview, a father complains that his adolescent son is staying out all night and is using drugs and alcohol. The father requests that the son be removed from the home immediately. The social worker should FIRST: refer the adolescent to a drug treatment program locate an alternative living situation for the adolescent recommend individual counseling You Answered You Answered recommend that the family begin family counseling Correct Answer Correct Answer More information is needed before any decisions are made, so the social worker should FIRST interview the adolescent and the family (key D). This is the important step in problem formulation. 1 / 1 pts Question 33 The stress experienced by a caregiver is more likely to result in elder abuse when the: caregiver is employed caregiver is not a relative
11/3/23, 10:36 PM MSW Practice Test: MSW Advanced Standing-2022 Fall Quarter 08/29-11/20 https://waldenu.instructure.com/courses/17315/quizzes/13999 14/65 older person is financially dependent impairment of the older person worsens Correct! Correct! Evidence demonstrates that the risk of elder abuse becomes greater as the mental and physical health of the older person begins to decline. Of the options given, (key D) is most likely to result in increased demands for care and subsequently place the older person at greater risk for abuse. 0 / 1 pts Question 34 Permanency planning is based on the value that every child has the right to a: choice of placement settings family setting Correct Answer Correct Answer long-term residential placement You Answered You Answered life plan in foster care The concept of permanency planning recognizes the need of every child to have a permanent home (key B) outside of foster care. 0 / 1 pts Question 35 A 60 -year-old man begins to exhibit an intense interest in his heritage. What does this behavior BEST represent? Life reflection as a part of the aging process Correct Answer Correct Answer An indication that he is experiencing identity issues You Answered You Answered Symptoms of disengagement from the larger society Signs of unresolved grief for the losses he has experienced (Key A) is correct because it represents a typical part of the older adulthood stage of life. None of the other options are indicated by the information in the stem. 0 / 1 pts Question 36 An older person with poor hearing may exhibit all of the following reactions EXCEPT: depression
11/3/23, 10:36 PM MSW Practice Test: MSW Advanced Standing-2022 Fall Quarter 08/29-11/20 https://waldenu.instructure.com/courses/17315/quizzes/13999 15/65 dementia Correct Answer Correct Answer paranoia You Answered You Answered isolation A person experiencing hearing loss is likely to experience options (A), (C), and (D) during the adjustment period as hearing worsens. (Key B) is a cognitive disorder unrelated to physical hearing. 0 / 1 pts Question 37 Which of the following assessment tools is the MOST effective for obtaining family history information? Person-in-Environment (PIE) System Social network mapping You Answered You Answered Genogram Correct Answer Correct Answer Ecomap A genogram (key C) is a social work tool that maps family relationships. PIE, option (A), may include explanations about the family and supports, but it is not an effective way to gather family history. (B) and (D) both identify client support systems. 0 / 1 pts Question 38 A parent meets with a social worker because of concerns about a child's behavior. The client's six -year-old child has recently begun hitting other children, yelling at the parent, and breaking toys. During the assessment, the client indicates that the child's other parent died two months earlier. Which action should the social worker take FIRST? Provide the parent with behavior management techniques Refer the child to a community anger management group You Answered You Answered Conduct a thorough depression assessment with the child Normalize the anger as a common part of the grieving process Correct Answer Correct Answer Young children often respond to loss with acting out behavior. While the other actions may be taken later, the FIRST thing a social worker should do is reassure the parent that this is most likely a normal response for the child in this circumstance (key D).
Your preview ends here
Eager to read complete document? Join bartleby learn and gain access to the full version
  • Access to all documents
  • Unlimited textbook solutions
  • 24/7 expert homework help
11/3/23, 10:36 PM MSW Practice Test: MSW Advanced Standing-2022 Fall Quarter 08/29-11/20 https://waldenu.instructure.com/courses/17315/quizzes/13999 16/65 1 / 1 pts Question 39 A social worker meets with a client who has difficulty eating because she fears that her food is contaminated. She repeatedly washes and checks her food. What disorder should the social worker FIRST suspect? Generalized anxiety Posttraumatic stress Obsessive-compulsive Correct! Correct! Bulimia nervosa Only (key C) is indicated by the information given. Answering the question requires a recalled knowledge of the symptoms of obsessive-compulsive disorder. 0 / 1 pts Question 40 A social worker and a lawyer are serving on an interdisciplinary team to provide and coordinate domestic violence services in the community. The social worker is obligated to obtain the consent of a client before releasing information. The lawyer is not required to obtain consent. This difference results in conflict between the social worker and the lawyer. What should the social worker do FIRST? Acknowledge to the team that confidentiality requirements vary by profession Correct Answer Correct Answer Seek modification of the team membership to exclude lawyers Divide the work of the team into legal and nonlegal functions Arrange training about differing professional ethical obligations for team members You Answered You Answered This is an unscored item. (Key A) would provide information to the team members, which may prevent further conflict and enhance collaboration. Option (B) is exclusionary rather than collaborative. Option (C) does not resolve the potential ethical dilemma and does not create a collaborative effort. There is not enough disagreement at this point to warrant (D). 0 / 1 pts Question 41 In social work practice, the MOST important reason for obtaining a client's social history is to provide a better understanding of the client and to determine: community resources which can provide needed services previous social work services used by the client details about the client's background You Answered You Answered
11/3/23, 10:36 PM MSW Practice Test: MSW Advanced Standing-2022 Fall Quarter 08/29-11/20 https://waldenu.instructure.com/courses/17315/quizzes/13999 17/65 services needed by the client Correct Answer Correct Answer Services needed (key D) is the answer because the social worker is initially focused on the needs of the client, and the social history is a means to determine that. (A), (B), and (C) may be components of the social history and may be helpful, but they are not the MOST important reason to obtain it. 0 / 1 pts Question 42 The power imbalance between the social worker and client in the assessment process is BEST minimized by: focusing on the client's understandings and goals Correct Answer Correct Answer seeking corroboration of client information from collateral sources giving direction to the client about the content of the assessment You Answered You Answered determining ownership of the assessment process This is an unscored item. (Key A) minimizes the power imbalance, as it is client focused. Seeking information about the client from outside sources enhances the power differential, as it does not involve the client. Option (C) perpetuates the social worker’s perceived power in this relationship. Option (D) does not reduce the power imbalance as the client may feel that the social worker is very much in control of the process. 0 / 1 pts Question 43 What is the BEST description of empathy? Communication of the social worker's desire to help Nonjudgmental acceptance of client behavior You Answered You Answered A compassionate feeling toward another person's suffering An insightful understanding of another person's thoughts and emotions Correct Answer Correct Answer This is another recall question, requiring the knowledge that empathy means that the social worker understands the emotions of the client without necessarily sharing them (key D). 0 / 1 pts Question 44 One of the MOST appropriate ways to demonstrate dignity and respect for older adult clients who hold traditional beliefs is to:
11/3/23, 10:36 PM MSW Practice Test: MSW Advanced Standing-2022 Fall Quarter 08/29-11/20 https://waldenu.instructure.com/courses/17315/quizzes/13999 18/65 address them by their last names and a title Correct Answer Correct Answer handle initial contacts in a familiar, informal way use sustained and direct eye contact You Answered You Answered proceed in the same way that one would with any other client group Until invited to do otherwise by the client, the social worker should address older clients in a more formal manner, e.g., “Mr. Smith.” Informality and sustained eye contact can be interpreted as disrespectful and would not necessarily acknowledge the unique characteristics of an older client population. 0 / 1 pts Question 45 In testifying at a court hearing, the social worker must: protect the client's confidentiality to the extent permitted by law Correct Answer Correct Answer provide a written summary of client contact to the court You Answered You Answered discuss the client's case freely when testifying allow the court to inspect the records (Key A) is the only situation that maintains client confidentiality at the court hearing. Options (B), (C), and (D) have the social worker revealing confidential information, which is not acceptable. 0 / 1 pts Question 46 A client requests help from a social worker to find safe and affordable housing. The client also wants to have the housing located in a certain area of town. How can the social worker BEST provide assistance while respecting the client's right to self-determination? Work with the client in formulating options that consider the client's preferences and financial status Correct Answer Correct Answer Refer the client to the local housing authority for information about public housing You Answered You Answered Contact several realty companies for information on apartments that might meet the client's budget Inform the client that the social worker will develop a housing plan before the next scheduled session (Key A) is correct because it involves the client in problem solving by allowing the client to take the lead. In options (B), (C), and (D), the social worker goes ahead with the approach that he or she thinks is best for the client.
Your preview ends here
Eager to read complete document? Join bartleby learn and gain access to the full version
  • Access to all documents
  • Unlimited textbook solutions
  • 24/7 expert homework help
11/3/23, 10:36 PM MSW Practice Test: MSW Advanced Standing-2022 Fall Quarter 08/29-11/20 https://waldenu.instructure.com/courses/17315/quizzes/13999 19/65 0 / 1 pts Question 47 The social work supervisor has noted an increase in the number of Muslim families served by the agency. The supervisor and unit workers have little knowledge of the Muslim religion. What should the supervisor do to better assist the Muslim families? Request Muslim families be assigned to Muslim workers only Arrange a meeting with the imam at the local mosque You Answered You Answered Arrange a mandatory training for the staff in religious diversity Correct Answer Correct Answer Locate a faith-based organization that would meet the needs of Muslim clients Social workers have an obligation to become culturally competent in their practice. A part of becoming competent and improving services to diverse groups is to learn about the groups represented in the clients seen at the agency. Options (A), (B), and (D) would do little to increase the cultural competence of the agency staff. 1 / 1 pts Question 48 When working with a client from a different cultural group, the social worker should FIRST: ignore the difference between the client and the worker learn about the client's cultural heritage Correct! Correct! treat the client the same as all other clients discuss the differences between the two cultures Part of cultural competence is for the social worker to learn about the client (key B). To ignore the difference, option (A), could lead to a poor working relationship. Option (C), treating all clients the same, is to ignore the social worker’s response to the individual. (D) Could be seen as intrusive early in the relationship. 1 / 1 pts Question 49 A five-year-old is having difficulty adjusting to kindergarten. The teacher has tried to help the child adjust but has had limited success. The teacher refers the child to the school social worker. The social worker should FIRST: establish a behavior contract with the child observe the child in the classroom Correct! Correct! refer the child to outside counseling
11/3/23, 10:36 PM MSW Practice Test: MSW Advanced Standing-2022 Fall Quarter 08/29-11/20 https://waldenu.instructure.com/courses/17315/quizzes/13999 20/65 call the family for a conference Prior to taking any of the actions mentioned in options (A), (C), and (D), the social worker must become directly familiar with the child’s behavior and with the teacher’s efforts in the classroom. 0 / 1 pts Question 50 During a session, a chronically depressed client mentions that "sometimes life does not seem worth living." At this point the MOST important task for the social worker is to: obtain permission to notify the client's family determine if the client has the means to commit suicide You Answered You Answered clarify what the client meant by the statement Correct Answer Correct Answer assist the client to focus on personal strengths The client’s statement may be indicative of suicidal ideation and, therefore, should be explored. This would be the first step in completing a suicide risk assessment in this situation 0 / 1 pts Question 51 The PRIMARY objective when beginning an interview is to: follow all specific policy guidelines establish control of the interview You Answered You Answered seek to interpret responses establish open communication Correct Answer Correct Answer It is crucial for a social worker to establish open communication in the initial interview. This allows for proper assessment of the client’s needs. Although policy guidelines must be followed, this is not the PRIMARY objective of any initial interview. It is never the objective for the social worker to establish control of the interview. Client responses can be interpreted only when there is open communication between the social worker and the client. 1 / 1 pts Question 52 A core value of the African American culture is:
11/3/23, 10:36 PM MSW Practice Test: MSW Advanced Standing-2022 Fall Quarter 08/29-11/20 https://waldenu.instructure.com/courses/17315/quizzes/13999 21/65 individualism atavism collectivism communalism Correct! Correct! Many African Americans ascribe to communalism (key D), a value that demonstrates a sense of responsibility toward the greater population than to the individual. The other three options are not typical values of African Americans. 0 / 1 pts Question 53 A social worker assists clients during their pregnancies as a component of a maternal health program. The social worker routinely provides assistance in accessing services. During a session, a client reports being diagnosed with preeclampsia and does not understand the diagnosis. What is the social worker's BEST course of action to assist the client? Suggest that the client speak with an obstetrician or another professional who has expertise in this area Correct Answer Correct Answer Provide answers to the client's questions based on the social worker's understanding of the diagnosis You Answered You Answered Explain to the client that the social worker is not an expert on the medical impact of conditions related to pregnancy Offer the client brochures and articles detailing information related to the health condition and its complications Even if the social worker has knowledge in this area, the client should be referred to another professional who works more directly with such issues. Option (C) is not helpful to the client, because it does not include a source for such information. 0 / 1 pts Question 54 A new law is passed that affects several agency clients. The social worker wishes to explain the law's changes and potential impact to those affected. What is the social worker's MOST effective course of action to accomplish this? Mail a copy of the law to each client Call each client to inform them of changes You Answered You Answered Display a list of changes in the clients' waiting area Discuss the changes in the law during the next session Correct Answer Correct Answer
Your preview ends here
Eager to read complete document? Join bartleby learn and gain access to the full version
  • Access to all documents
  • Unlimited textbook solutions
  • 24/7 expert homework help
11/3/23, 10:36 PM MSW Practice Test: MSW Advanced Standing-2022 Fall Quarter 08/29-11/20 https://waldenu.instructure.com/courses/17315/quizzes/13999 22/65 (Key D) is the best option because it allows the client to ask any questions or obtain clarification as needed. While this could be done by telephone, this would not be as efficient. Mailing a copy of the law would not assist clients who have difficulty reading. Options (A) and (C) do not guarantee that clients will become familiar with the information. 1 / 1 pts Question 55 A social worker provides case management services to a family recently evicted from their apartment. The social worker gives the family a list of available temporary housing. This action is an example of: networking empowerment Correct! Correct! enabling feedback The social worker is empowering the clients by providing them with information about available resources and encouraging them to make the contact themselves. The definitions of the other options do not apply in this situation. 0 / 1 pts Question 56 Which statement would accurately represent effective, ethical case recording? The client's neighborhood is labeled as a slum. The client's parents are called "Mom" and "Dad" in the record. You Answered You Answered The client is described as "a 25 -year-old male." Correct Answer Correct Answer The client is represented as "a lazy 19 -year-old boy." Options (A) and (D) use subjective judgment that is not ethical. Option (B) is not an appropriate way to identify the clients in the case recording. (Key C) is objective, fact-based, and does not include any opinions or judgments by the social worker. 0 / 1 pts Question 57 In social work with groups, the PRIMARY task of the facilitator is to further the group process toward what end? Goal achievement Correct Answer Correct Answer
Your preview ends here
Eager to read complete document? Join bartleby learn and gain access to the full version
  • Access to all documents
  • Unlimited textbook solutions
  • 24/7 expert homework help
11/3/23, 10:36 PM MSW Practice Test: MSW Advanced Standing-2022 Fall Quarter 08/29-11/20 https://waldenu.instructure.com/courses/17315/quizzes/13999 23/65 Group preaffiliation You Answered You Answered Role differentiation Norm development This is an unscored item. (Key A) is the purpose of any social work service. Options (B) and (C) are elements of group dynamics, while option (D) is a stage in group work. 0 / 1 pts Question 58 A child's physical appearance and intelligence are repeatedly criticized by peers and family members. The parents provide little physical affection and affirmation. They also refuse to intervene on the child's behalf. The child is MOST likely a victim of which type of maltreatment? Psychological abuse Correct Answer Correct Answer Physical neglect You Answered You Answered Physical abuse Medical neglect Psychological abuse (key A) typically involves maligning an individual and inflicting emotional stress. 0 / 1 pts Question 59 A social service agency develops a form to be used by the staff that will document steps completed by clients to achieve desired outcomes. Use of the form is an example of which of the following tasks? Monitoring the intervention process Correct Answer Correct Answer Recording systematic information You Answered You Answered Implementing change oriented strategies Comparing baseline data with behavior change The purpose of the form is to record actions the client has taken in working toward achievement of the identified goals (key A). As such, it serves a monitoring function. 0 / 1 pts Question 60
Your preview ends here
Eager to read complete document? Join bartleby learn and gain access to the full version
  • Access to all documents
  • Unlimited textbook solutions
  • 24/7 expert homework help
11/3/23, 10:36 PM MSW Practice Test: MSW Advanced Standing-2022 Fall Quarter 08/29-11/20 https://waldenu.instructure.com/courses/17315/quizzes/13999 24/65 A social worker sees a family with an alcoholic father. The adolescent son, in contrast to his younger siblings, is clean, well dressed, and doing well in school. During the interview, he interrupts his mother several times to explain "what she means." The child has probably assumed which of the following roles in his family system? Hero Correct Answer Correct Answer Enabler You Answered You Answered Mascot Peacemaker This is a recall question that requires knowledge of the roles in a family system that includes an alcoholic. The hero in this type of family (key A) exhibits behaviors that are positive, mature, and rescuing toward the other family members. This is seen in the teenage son’s behaviors. The enabler would create an environment that would allow the alcoholic to continue drinking. The mascot would provide humorous experiences for the family. The peacemaker would do whatever was necessary to avoid conflict and maintain peace in the household. 0 / 1 pts Question 61 Renowned African American neurosurgeon Ben Carson grew up in a one-parent home, headed by his mother who was illiterate and extremely poor. To conclude that Dr. Carson's achievements are more typical of middle- and upper-class expectations suggests: racial typecasting stereotypical thinking Correct Answer Correct Answer intergenerational mobility You Answered You Answered institutional racism (Key B) is correct because stereotypical thinking is based on overgeneralization, in this case expected behavior. This is not racial typecasting (A) because the question asks about class expectations rather than racial expectations. There is nothing in the stem to indicate that either option (C) or option (D) are applicable to a question about class. 1 / 1 pts Question 62 A child care worker discovers a five -year-old girl peeking under a toilet stall at a five -year-old boy. Which of the following statements BEST explains the girl's behavior? She has an age appropriate curiosity about bodily functions and gender differences in anatomy. Correct! Correct! She needs to be observed closely to assess her risk for sexual perversion.
Your preview ends here
Eager to read complete document? Join bartleby learn and gain access to the full version
  • Access to all documents
  • Unlimited textbook solutions
  • 24/7 expert homework help
11/3/23, 10:36 PM MSW Practice Test: MSW Advanced Standing-2022 Fall Quarter 08/29-11/20 https://waldenu.instructure.com/courses/17315/quizzes/13999 25/65 She is acting out as a result of premature exposure to sexual behavior. She has not successfully resolved the Electra complex. This behavior (key A) is age appropriate for a child of five years. There is no information in the stem to indicate that options (B), (C), or (D) would be applicable in this situation. 0 / 1 pts Question 63 During a presentation on child development, a parent asks a social worker the age at which a child begins to run. What is the MOST appropriate response? 10 to 11 months 1 to 1.5 years You Answered You Answered 2 to 3 years Correct Answer Correct Answer 3.5 to 4.5 years According to developmental models, children are developmentally on target to develop gross motor coordination, including the ability to run, between the ages of 2 to 3 years (key C). This is a recall question. 0 / 1 pts Question 64 A community social service agency receives funding to offer group HIV education programs. In consultation with the supervisor, the social worker realizes that a group setting for HIV education may not be the best approach to use. The program goals require that high numbers of clients be served. What should the social worker do FIRST? Continue to offer group educational sessions Begin to offer only individual educational sessions You Answered You Answered Formulate a client committee to develop recommendations Correct Answer Correct Answer Return the money to the funding source (Key C) is correct because it allows the client population to identify the problem, if there is one, before immediately going to a solution. The social worker may do the other options, but (key C) is the FIRST step. Of course, option (D) is very unlikely until all possible ways to use the funding are carefully considered and even tried. 1 / 1 pts Question 65
Your preview ends here
Eager to read complete document? Join bartleby learn and gain access to the full version
  • Access to all documents
  • Unlimited textbook solutions
  • 24/7 expert homework help
11/3/23, 10:36 PM MSW Practice Test: MSW Advanced Standing-2022 Fall Quarter 08/29-11/20 https://waldenu.instructure.com/courses/17315/quizzes/13999 26/65 After promoting a new support group for single parents for many weeks before starting the group, a social worker is discouraged and wants to understand the reasons for poor attendance in the first two sessions. To gain a better understanding of the situation, the social worker should FIRST: change the promotion tactics for the support group to reach a broader group of potential participants conduct a needs assessment to determine if neighborhood residents view the support group as a needed service Correct! Correct! interview other professionals for advice on how to improve the content of the support group involve community leaders who might encourage potential participants to attend the support group Lack of attendance at the start of a well-publicized program may be an indication that the program is unnecessary. This should be determined before other options are considered. 1 / 1 pts Question 66 Which model MOST often uses ecomaps and genograms? Functional Psychosocial Correct! Correct! Existential Psychoanalytic The psychosocial model (key B) of practice places emphasis on the use of ecomaps and genograms to gain an understanding of the person-in-environment perspective. This emphasis is not seen in the other three approaches to practice. 0 / 1 pts Question 67 A school social worker is interviewing a ten-year-old boy who has recently been placed in foster care due to maltreatment. His teachers describe an increase in aggressive and anxious behaviors. What is the BEST explanation for the child’s reactions? School phobia Reaction formation You Answered You Answered Traumatic stress Correct Answer Correct Answer Learning disabilities
Your preview ends here
Eager to read complete document? Join bartleby learn and gain access to the full version
  • Access to all documents
  • Unlimited textbook solutions
  • 24/7 expert homework help
11/3/23, 10:36 PM MSW Practice Test: MSW Advanced Standing-2022 Fall Quarter 08/29-11/20 https://waldenu.instructure.com/courses/17315/quizzes/13999 27/65 It is not unusual for a child who has experienced a series of traumatic events (key C) in a short period to react with anxiety and aggression. There is no indication that he has school phobia or learning disabilities. The use of reaction formation as a defense mechanism is not evident. 0 / 1 pts Question 68 Which value statement MOST closely represents a client's freedom to question a social worker's intervention? The client's right to self-determination Correct Answer Correct Answer The intrinsic worth and value of the individual You Answered You Answered The client's right to share the benefits of society Mutual rights and responsibilities for all Options (B), (C), and (D) are all guiding principles of social work practice but do not directly relate to clients’ right to question a social worker’s intervention. (Key A) takes into account clients’ right to make choices for their own best interest, which may mean choosing not to participate in a particular intervention or to question its use 0 / 1 pts Question 69 The PRIMARY focus of supervisory activity for social work students should be: integrating theoretical knowledge with practice experience Correct Answer Correct Answer demonstrating that the student understands professional ethics assuring that students can use assessment tools You Answered You Answered helping the student use reflection skills While options (B), (C), and (D) can be a part of a social work student’s training, the PRIMARY task of supervision is to help the student integrate theory and practice skills. 0 / 1 pts Question 70 Unanswered Unanswered A social worker learns that a client's partner abuses alcohol. The client frequently calls the partner's employer to make excuses for work absences. The client often takes primary responsibility for parenting and household duties when the partner is recovering from alcohol binges. How should the social worker respond to these concerns? Explore the role the client plays in supporting unhealthy family patterns Correct Answer Correct Answer
Your preview ends here
Eager to read complete document? Join bartleby learn and gain access to the full version
  • Access to all documents
  • Unlimited textbook solutions
  • 24/7 expert homework help
11/3/23, 10:36 PM MSW Practice Test: MSW Advanced Standing-2022 Fall Quarter 08/29-11/20 https://waldenu.instructure.com/courses/17315/quizzes/13999 28/65 Offer community resource referrals for parenting support and child care Provide information about the local Alcoholics Anonymous organization Refer the client to a physician to rule out medical reasons for fatigue The social worker should begin with (key A) because that is starting where the client is to offer the most immediate help through increased understanding by the client. The social worker might then do options (B) and (C), with (D) a possibility if fatigue emerges as a problem. 0 / 1 pts Question 71 Unanswered Unanswered A social worker in an assisted living facility learns that a resident is being financially exploited by the staff. The social worker reports this to the facility director, who refuses to take any action. Ethically, what should the social worker do NEXT? Report the facility to a regulatory board Investigate the situation independently Inform the director of the social worker's obligation to report Correct Answer Correct Answer Report the problem to the resident's family According to the social work code of ethics, the social worker is mandated to report abuse and neglect, including financial exploitation, but must also take steps to be sure employers or employees are aware of the obligation (key C). The social worker has already conveyed the suspicions to the facility director. Before going to the next level, option (A), (key C) must be done. Reporting to the resident’s family, option (D), would not be relevant 0 / 1 pts Question 72 Unanswered Unanswered A hospital social worker is interviewing the parents of a child who has juvenile diabetes. The mother accuses the father of ignoring the child's medical condition, and the father accuses the mother of overprotecting the child. The parents' behavior is MOST likely an expression of: misunderstanding of the child's diagnosis anger related to the parents' marital discord differences in the parents' personalities the parents' frustration with the child's diagnosis Correct Answer Correct Answer
Your preview ends here
Eager to read complete document? Join bartleby learn and gain access to the full version
  • Access to all documents
  • Unlimited textbook solutions
  • 24/7 expert homework help
11/3/23, 10:36 PM MSW Practice Test: MSW Advanced Standing-2022 Fall Quarter 08/29-11/20 https://waldenu.instructure.com/courses/17315/quizzes/13999 29/65 Although the parents disagree about managing the child’s medical situation, there is no evidence in the stem of major marital problems, personality con±ict, or misunderstanding of the child’s illness. Having a child with a serious chronic disease can cause frustration and con±ict in the family system. 0 / 1 pts Question 73 Unanswered Unanswered A single mother informs the social worker of her intention to relocate without notifying the father of her children. The client is estranged from her immediate family and has been having relationship problems with the children's father. The social worker's PRIMARY focus in the initial interview should be to: remind the client that contact with the father is important to the welfare of the children inform the client that the local child support services unit will have to be notified of her plans assist the client in notifying the father of her intentions discuss with the client the issues leading to her decision Correct Answer Correct Answer This is an unscored item. Option (A) is not empowering and does not give priority to the client’s wishes. Although that information may be important, it would not be the PRIMARY focus of the initial interview. It is not necessary to inform child support services of the client’s intention to move. Option (C) would not be an appropriate action to take until the client’s reason for her decision is discussed. 0 / 1 pts Question 74 Unanswered Unanswered Which activity would BEST fulfill a social worker’s ethical responsibility to promote the general welfare of society? Advocating for changes to social policy issues Correct Answer Correct Answer Reporting suspected child abuse as mandated Attending continuing education programs on ethics Volunteering on agency boards Although options (B), (C), and (D) are activities that can either directly or indirectly benefit clients, only (key A) promotes the general welfare of society. 0 / 1 pts Question 75 Unanswered Unanswered A group home resident who is developmentally disabled has been exhibiting inappropriate behavior. To use behavior modification with this client, the social worker should FIRST: ask all group home staff members for their observations
Your preview ends here
Eager to read complete document? Join bartleby learn and gain access to the full version
  • Access to all documents
  • Unlimited textbook solutions
  • 24/7 expert homework help
11/3/23, 10:36 PM MSW Practice Test: MSW Advanced Standing-2022 Fall Quarter 08/29-11/20 https://waldenu.instructure.com/courses/17315/quizzes/13999 30/65 elicit the support of the resident’s parents call the day program staff and ask whether the behavior occurs there define the target behavior and develop a baseline Correct Answer Correct Answer This is an unscored item. A basic step in using behavior modification principles to alter behavior involves specifying the targeted behavior and its frequency (key D). An understanding of the behavior is required before a service plan can be developed. 0 / 1 pts Question 76 Unanswered Unanswered A social worker and a client who are able to communicate with mutual understanding and without great conflict MOST likely have established: rapport Correct Answer Correct Answer adjustment cooperation countertransference This is an unscored item. Rapport is based on compatibility between the social worker and the client. It fosters mutual understanding and a strong working relationship between the two. Options (B), (C), and (D) are not applicable in this situation. 0 / 1 pts Question 77 Unanswered Unanswered The PRIMARY benefit of a multidisciplinary approach to social problems is to: deliver services economically deliver more effective services Correct Answer Correct Answer foster effective collaboration encourage centralization of programs As actions that accompany a multidisciplinary approach, options (A), (C), and (D) are beneficial to the social workers, agencies, and programs involved. However, the PRIMARY benefit is to create services that are more effective in meeting clients’ needs. 0 / 1 pts Question 78 Unanswered Unanswered
Your preview ends here
Eager to read complete document? Join bartleby learn and gain access to the full version
  • Access to all documents
  • Unlimited textbook solutions
  • 24/7 expert homework help
11/3/23, 10:36 PM MSW Practice Test: MSW Advanced Standing-2022 Fall Quarter 08/29-11/20 https://waldenu.instructure.com/courses/17315/quizzes/13999 31/65 Which of the following interviewing skills is used in social work to help the client set treatment goals? Containment Focusing Correct Answer Correct Answer Summarizing Reassuring This is a recall question. Focusing (key B) is helping the client define reasons for seeking services and defining treatment goals. 0 / 1 pts Question 79 Unanswered Unanswered A social worker in a senior citizens center has become aware of an elderly couple's inability to maintain themselves. They live in a rural area without any nearby family members and are unwilling to consider alternative living arrangements. The social worker should FIRST: try to persuade the couple to go to a nursing home discuss referral for supportive services Correct Answer Correct Answer inform family members of the situation arrange for a mental status exam to be completed The social worker will support self-determination by using the least restrictive and controlling option FIRST (key B). Option (C) is a violation of confidentiality unless there is real endangerment at issue. 0 / 1 pts Question 80 Unanswered Unanswered A social worker is sensing what the client is feeling and communicates understanding of the feelings back to the client. The social worker is using: warmth empathy Correct Answer Correct Answer genuineness respect
Your preview ends here
Eager to read complete document? Join bartleby learn and gain access to the full version
  • Access to all documents
  • Unlimited textbook solutions
  • 24/7 expert homework help
11/3/23, 10:36 PM MSW Practice Test: MSW Advanced Standing-2022 Fall Quarter 08/29-11/20 https://waldenu.instructure.com/courses/17315/quizzes/13999 32/65 This is an unscored item. Options (A), (C), and (D) are not social work tools used in interactions with clients. Empathy (key B) allows the social worker to convey an understanding of the client’s feelings and of the circumstances that prompted the feelings. 0 / 1 pts Question 81 Unanswered Unanswered A child who has been physically abused will typically display all of the following behaviors EXCEPT: protectiveness of abuser attraction to strangers Correct Answer Correct Answer fear of sudden movements withdrawal This is a recall question, requiring knowledge of the behavior of an abused individual. 0 / 1 pts Question 82 Unanswered Unanswered A social worker in child protective services contacts a mother reported for neglect of her seven-year-old son. The social worker should FIRST gather information for decision-making by: observing household environment for safety risks interviewing the child and mother, separately and together reading the current report of suspected neglect Correct Answer Correct Answer obtaining collateral data on the mother from neighbors The social worker should begin with existing objective data (key C) before proceeding with the investigation. Options (A), (B), and (D) are additional steps to take but would not be FIRST. 0 / 1 pts Question 83 Unanswered Unanswered A client meets with a social worker because the client's spouse is physically violent and emotionally abusive. The client expresses that she is trapped because the couple's religious beliefs prohibit divorce. How should the social worker FIRST respond to the client's concern? Offer to facilitate a meeting with religious leaders to discuss the situation Explore further the importance of religion in the client's life Correct Answer Correct Answer
Your preview ends here
Eager to read complete document? Join bartleby learn and gain access to the full version
  • Access to all documents
  • Unlimited textbook solutions
  • 24/7 expert homework help
11/3/23, 10:36 PM MSW Practice Test: MSW Advanced Standing-2022 Fall Quarter 08/29-11/20 https://waldenu.instructure.com/courses/17315/quizzes/13999 33/65 Provide the client with a referral to a domestic violence support group Challenge the client's choice to value religion more than personal safety This is an example of “starting where the client is,” with (key B). A referral might be appropriate for the client later, but it would not be the FIRST thing the social worker would do. Option (A) would not be appropriate unless the client requested the meeting. Option (D) discounts the importance the client places on her religious beliefs. 0 / 1 pts Question 84 Unanswered Unanswered A lawyer obtains a court order instructing a social worker to release case records from interviews with a client. The social worker is of the opinion that releasing the records would not be in the best interest of the client. The social worker should: notify the supervisor of the request from the lawyer Correct Answer Correct Answer advise the client to give permission to provide the record remove the harmful information in the record and then release it refuse to relinquish the records to protect client confidentiality It is important that the supervisor is made aware of the legal request in order to provide direction to the social worker. If releasing the records would not be in the client’s best interest, option (B) could cause undue harm to the client, which would be unethical. The social worker may not have the legal right to remove any information or to refuse to relinquish the record. Discussing the request with the supervisor is necessary 0 / 1 pts Question 85 Unanswered Unanswered A terminally ill client is concerned about being placed on life support and is unsure whether his family could make the decision to turn off life support if there is no chance of recovery. The social worker should FIRST: explore the client's religious faith suggest the client hold a family conference refer the client to the local hospice arrange for the client to discuss advance directives Correct Answer Correct Answer (Key D) is the most empowering action, as it allows the client to discuss any wishes or options and to make informed choices. While options (A) and (C) may become part of the intervention process at some point, they would not be the FIRST actions as they do not address the client’s concern of having life support stopped. Option (B) would take place after the client has fully discussed any advance directive with the appropriate professionals.
Your preview ends here
Eager to read complete document? Join bartleby learn and gain access to the full version
  • Access to all documents
  • Unlimited textbook solutions
  • 24/7 expert homework help
11/3/23, 10:36 PM MSW Practice Test: MSW Advanced Standing-2022 Fall Quarter 08/29-11/20 https://waldenu.instructure.com/courses/17315/quizzes/13999 34/65 0 / 1 pts Question 86 Unanswered Unanswered In developing a research project, what should the social worker do FIRST? Recruit people to work on the project Review the literature on the topic Write a proposal for funding Define the research problem Correct Answer Correct Answer This is an unscored item. The FIRST step in any research project is to define the research question. This question will guide the literature review. Proposals for funding and recruitment of persons to work on the project are later steps in the process. 0 / 1 pts Question 87 Unanswered Unanswered A woman sees a social worker because she feels unable to fulfill her roles of mother and employee to her satisfaction. She is experiencing role: ambiguity conflict Correct Answer Correct Answer discontinuity reversal The demands of the two roles are in conflict with each other (key B). Performing well in one role can be detrimental to the woman’s performance in the other role. In contrast, role ambiguity, option (A), is a situation in which role expectations are not clear. 0 / 1 pts Question 88 Unanswered Unanswered A client states, "I would like your opinion about how I reacted to my boss asking me to work overtime again this week." In response, the social worker should use the technique of: reflection stress management feedback Correct Answer Correct Answer universalization
Your preview ends here
Eager to read complete document? Join bartleby learn and gain access to the full version
  • Access to all documents
  • Unlimited textbook solutions
  • 24/7 expert homework help
11/3/23, 10:36 PM MSW Practice Test: MSW Advanced Standing-2022 Fall Quarter 08/29-11/20 https://waldenu.instructure.com/courses/17315/quizzes/13999 35/65 The social worker would be providing feedback (key C) when offering an opinion about a specific action, which is what the client is requesting. Option (A), reflection, enables clients to think about their behaviors or perceptions. The client’s request is not an example of stress management, option (B), nor is it an example of universalization, option (D), which would be discussing ways in which the experience with the boss would apply to everyone. Again, the client is asking about a specific action. 0 / 1 pts Question 89 Unanswered Unanswered In an initial interview with a social worker, a client requests assistance with transportation for a family member undergoing medical treatment. The agency has no provision for offering assistance with transportation needs. The social worker should: try to meet the transportation need personally on a short-term basis explain to the client that this request does not fall within agency function contact the physician to discuss alternative treatment locations refer the client to another agency offering transportation assistance Correct Answer Correct Answer It would not be appropriate for the social worker to provide transportation, even if the agency allows this practice. Explaining the agency function does not help the client, as it leaves the client without services. It would not be appropriate to discuss alternative treatment locations as it may not be possible in that area and could prevent the client from obtaining the proper medical care. The only appropriate response is to refer the client to an agency that can provide transportation services. 0 / 1 pts Question 90 Unanswered Unanswered A mother and her two children have been referred to a social worker. The mother has just left an abusive relationship with the children's father, and she and the children are living in a shelter. The social worker should FIRST: refer the mother to a domestic violence program determine the safety and well-being of the children respond to the family's most immediate needs Correct Answer Correct Answer analyze the resources of the family's social network (Key C) begins where the client is, which is most important in this situation. Options (A), (B), and (D) may be relevant once the social worker has an understanding of the situation. 0 / 1 pts Question 91 Unanswered Unanswered
Your preview ends here
Eager to read complete document? Join bartleby learn and gain access to the full version
  • Access to all documents
  • Unlimited textbook solutions
  • 24/7 expert homework help
11/3/23, 10:36 PM MSW Practice Test: MSW Advanced Standing-2022 Fall Quarter 08/29-11/20 https://waldenu.instructure.com/courses/17315/quizzes/13999 36/65 In talking with a client, a social worker asks specific questions about the current date, mood, and memory. The social worker is collecting data to assess the client's: interpersonal skills intellectual ability tolerance level mental status Correct Answer Correct Answer These questions asked by the social worker are used as a part of a mental status exam (key D). Collection of data on the other three characteristics would be assessed through other means. 0 / 1 pts Question 92 Unanswered Unanswered Structuring a social work interview means: identifying the case problems in the social worker's report clarifying the roles of the social worker and the client Correct Answer Correct Answer bringing out the emotional problems of the client arranging the furniture in the interview room Structuring is done at the initial phase to set client expectations for work to be done and the ways in which the social worker and the client will interact (key B). This way, the client’s understanding of the roles is the same as the social worker’s. 0 / 1 pts Question 93 Unanswered Unanswered Which is the MOST distinctive indicator of child physical abuse? A reddened area on the legs or arms Bruises in the shape of objects Correct Answer Correct Answer A broken leg, arm, or foot Frequent crying and sadness Options (A), (C), and (D) may be indicators of abuse and may require further investigation, but any of these injuries may occur with normal childhood activities. (Key B) is the MOST distinctive.
Your preview ends here
Eager to read complete document? Join bartleby learn and gain access to the full version
  • Access to all documents
  • Unlimited textbook solutions
  • 24/7 expert homework help
11/3/23, 10:36 PM MSW Practice Test: MSW Advanced Standing-2022 Fall Quarter 08/29-11/20 https://waldenu.instructure.com/courses/17315/quizzes/13999 37/65 0 / 1 pts Question 94 Unanswered Unanswered A chronically ill child is hospitalized in critical condition. The child’s parents initiate a religious ceremony at the child’s bedside that includes a religious leader and extended family members. Hospital staff observing this ritual ask that everyone leave the room. The family members become upset and refuse to leave the child. The nursing supervisor asks the social worker to intervene. The social worker should FIRST: educate the ward staff about the family’s cultural and religious beliefs Correct Answer Correct Answer explore the nature of the parents’ anxiety about the child’s condition ask the parents to explain the significance of the religious service explain hospital policy to the parents regarding the number of persons allowed in a room (Key A) provides an opportunity for the social worker to serve as an advocate for the family as well as creating an opportunity to eliminate discrimination based on the staff’s misunderstanding of cultural differences. Because the family is facing a critical situation with the child, options (B), (C), and (D) would not be appropriate actions at this time. 0 / 1 pts Question 95 Unanswered Unanswered A social worker is conducting an interview with a client whose race differs from that of the social worker. To develop rapport, the social worker should FIRST: examine self-attitudes and behaviors for possible biases Correct Answer Correct Answer acknowledge with the client any existing cultural differences appreciate human differences and experience seek consultation about the client’s racial group The FIRST step in becoming a culturally competent social worker is to honestly identify and assess any personal biases the social worker may have. The other options may be helpful in developing rapport with the client, but none of them would be the FIRST action to take. 0 / 1 pts Question 96 Unanswered Unanswered A group of tenants has asked for assistance in improving the physical condition of a public housing unit. The social worker should FIRST: independently assess the extent of the problem make the local housing authority aware of the problem
Your preview ends here
Eager to read complete document? Join bartleby learn and gain access to the full version
  • Access to all documents
  • Unlimited textbook solutions
  • 24/7 expert homework help
11/3/23, 10:36 PM MSW Practice Test: MSW Advanced Standing-2022 Fall Quarter 08/29-11/20 https://waldenu.instructure.com/courses/17315/quizzes/13999 38/65 organize a meeting of all tenants to discuss the problem and possible strategies Correct Answer Correct Answer assist the tenants in obtaining the materials necessary to make the improvements The social worker must begin by understanding the tenants’ concerns. Once this is accomplished, the tenants should be involved in finding an appropriate resolution to their problem. 0 / 1 pts Question 97 Unanswered Unanswered A client who is gay meets with a social worker to address symptoms of depression. The social worker becomes aware of having strong negative feelings about the client's sexual orientation. What should the social worker do in response to the feelings? Refer the client to another social worker Address the feelings with the client Discuss the feelings with the supervisor Correct Answer Correct Answer Work with the client despite the feelings Of the possible answers, (key C) is correct because a principal responsibility of the social worker is to understand the importance of value neutrality. The feelings need to be worked through with the supervisor. Options (A) and (B) put the social worker’s values above the role of serving the client. Option (D) ignores the problem and is potentially damaging to the client. 0 / 1 pts Question 98 Unanswered Unanswered In the initial phase of the problem-solving process, social workers apply their knowledge and skill to help clients: negotiate their social environment understand how their past influenced their present situation explore and define a target problem Correct Answer Correct Answer plan to maintain gains Of the options presented, (key C) is the correct response. The problem must be identified before problem- solving efforts are initiated. The remaining options are seen in later stages of the problem-solving process. 0 / 1 pts Question 99 Unanswered Unanswered
Your preview ends here
Eager to read complete document? Join bartleby learn and gain access to the full version
  • Access to all documents
  • Unlimited textbook solutions
  • 24/7 expert homework help
11/3/23, 10:36 PM MSW Practice Test: MSW Advanced Standing-2022 Fall Quarter 08/29-11/20 https://waldenu.instructure.com/courses/17315/quizzes/13999 39/65 While a social worker is facilitating a weekly support group, a client becomes very disruptive. The social worker should: ignore the disruptive behavior and speak to the client about the behavior after the group survey group members and decide how to respond based on their responses to the behavior ask group members to tell the client how they are being affected by the behavior Correct Answer Correct Answer stop the group and confront the client on the behavior in front of the members According to group intervention theory, allowing the group to respond to disruptive behavior (key C) assists in group cohesiveness and is part of the stages of group development. 0 / 1 pts Question 100 Unanswered Unanswered A client has a partner who is addicted to drugs. The client frequently assists the partner with obtaining prescription narcotics by giving false information to physicians. Which statement BEST describes the client's behavior? The client is violating the partner's confidentiality. The client is acting out of concern for the partner's health. The client is attempting to preserve the relationship. The client has underlying issues of codependency. Correct Answer Correct Answer Codependent behavior permits the partner’s drug use to continue while meeting some needs of the client. Confidentiality has not been violated, and the codependent behavior would not continue if the client is concerned about the partner’s health. There is nothing to indicate that the intent is to preserve the relationship by continuing with these behaviors. 0 / 1 pts Question 101 Unanswered Unanswered A man is frequently angry at his wife and does not let her know of his feelings. Instead, he explodes with anger at his adolescent son. According to ego psychology, which defense mechanism is demonstrated by the man's behavior? Reaction formation Denial Projection Displacement Correct Answer Correct Answer
Your preview ends here
Eager to read complete document? Join bartleby learn and gain access to the full version
  • Access to all documents
  • Unlimited textbook solutions
  • 24/7 expert homework help
11/3/23, 10:36 PM MSW Practice Test: MSW Advanced Standing-2022 Fall Quarter 08/29-11/20 https://waldenu.instructure.com/courses/17315/quizzes/13999 40/65 Ego psychology indicates that the placing of feelings about one person onto another is defined as displacement (key D). 0 / 1 pts Question 102 Unanswered Unanswered A traditional Native American/First Nations elder is seeking housing assistance. The social worker does not want to be perceived as intrusive during their meeting. What is the BEST way to achieve this? Ask the client to explain tribal culture Avoid asking direct questions of the client Speak softly in order to show respect to the client Avoid unnecessary direct eye contact with the client Correct Answer Correct Answer Option (A) is intrusive and inappropriate. Option (B) will not provide the necessary information required during the meeting. Option (C) does not actually show respect. (Key D) is the only action that shows respect for traditional Native American/First Nations culture. 0 / 1 pts Question 103 Unanswered Unanswered A social worker is providing case management services to a client. At one of their sessions, the client expresses the intent to join a local support group for widowed persons. The social worker is an active participant in the group. How can the social worker ethically handle this situation? Inform the client of the social worker’s membership Correct Answer Correct Answer Address the issue when the client attends a meeting Refer the client to another social worker for management Discourage membership in the support group The situation has the potential of developing into a dual relationship. It is the social worker’s ethical responsibility to establish appropriate professional boundaries so this will not occur. Knowing about the social worker’s membership in the group enables the client to make a decision about joining the group. 0 / 1 pts Question 104 Unanswered Unanswered When first interviewing a family, a social worker will make the family comfortable by: explaining the purpose of the interview Correct Answer Correct Answer
Your preview ends here
Eager to read complete document? Join bartleby learn and gain access to the full version
  • Access to all documents
  • Unlimited textbook solutions
  • 24/7 expert homework help
11/3/23, 10:36 PM MSW Practice Test: MSW Advanced Standing-2022 Fall Quarter 08/29-11/20 https://waldenu.instructure.com/courses/17315/quizzes/13999 41/65 asking family members about themselves self-disclosing asking the family members to explain the purpose of the interview (Key A) is the best way to reduce anxiety about what is going to take place. It clarifies the roles of the participants and starts the interview on a neutral basis. Option (B) is intrusive and would not ease the family’s possible discomfort. It would not be appropriate for the social worker to self-disclose at this point in the interview. It is not the family’s role to explain the purpose of the interview. 0 / 1 pts Question 105 Unanswered Unanswered Which of the following situations BEST indicates that a client is ready for termination of social work services? The client is consistently noncompliant with the service plan. The service contract is satisfactorily completed. Correct Answer Correct Answer The social worker is angry with the client. The client is making very slow progress. This is an unscored item. Options (A), (C), and (D) would not justify termination of services with the client. The service plan should include client goals and means for goal attainment. Once these are met, the client is ready for termination unless new goals are negotiated. 0 / 1 pts Question 106 Unanswered Unanswered A 67-year-old client is involved in a program designed to help participants upgrade their skills through on-the-job training. The client contacts the program’s social worker to report an inability to perform work because it is too difficult. The client wants to be transferred to another job site but does not want to confront the supervisor with concerns. What should the social worker do FIRST? Discuss positive aspects of the job with the client Assist the client in developing the next steps Correct Answer Correct Answer Approach the client’s supervisor on behalf of the client Review the job description with the client This is an unscored item. Development of next steps can be empowering to the client, as it will identify strengths that the client can use in problem resolution. If the job is too difficult for the client, other job possibilities need to be explored. This makes options (A) and (D) inappropriate. To approach the client’s supervisor would not be useful, since the client wants to be transferred.
Your preview ends here
Eager to read complete document? Join bartleby learn and gain access to the full version
  • Access to all documents
  • Unlimited textbook solutions
  • 24/7 expert homework help
11/3/23, 10:36 PM MSW Practice Test: MSW Advanced Standing-2022 Fall Quarter 08/29-11/20 https://waldenu.instructure.com/courses/17315/quizzes/13999 42/65 0 / 1 pts Question 107 Unanswered Unanswered In the evaluation of a client's capacity to change impulsive behavior, it is MOST important to assess the client's: self-esteem perceptiveness ability to delay gratification motivation Correct Answer Correct Answer Motivation for change is the only factor listed that directly influences a person’s capacity for change. The other factors do not necessarily influence this capacity. 0 / 1 pts Question 108 Unanswered Unanswered Process recordings are MOST appropriately used for: evaluating social worker-client interaction Correct Answer Correct Answer determining amount of program funding establishing client's eligibility for service determining social worker accountability Process recordings are case notes that provide information relating to the interaction between the social worker and the client. Options (B) and (D) are not connected with process recording, as they are not related to the client. Option (C) does not discuss the process of the interaction and would involve a different type of recording 0 / 1 pts Question 109 Unanswered Unanswered Which of the following components are MOST likely to be included in a social history? Appearance and attitude Mood and affect Perceptual distortions Education and work experience Correct Answer Correct Answer
Your preview ends here
Eager to read complete document? Join bartleby learn and gain access to the full version
  • Access to all documents
  • Unlimited textbook solutions
  • 24/7 expert homework help
11/3/23, 10:36 PM MSW Practice Test: MSW Advanced Standing-2022 Fall Quarter 08/29-11/20 https://waldenu.instructure.com/courses/17315/quizzes/13999 43/65 A social history contains factual information that includes education and work experience. Options (A), (B), and (C) represent subjective information based on the social worker’s observations during the interview. 0 / 1 pts Question 110 Unanswered Unanswered A social worker is working with a hostile client who continually arrives late for sessions. Despite discussions about the agency regulation which requires that clients more than 15 minutes late be rescheduled, the client demands the full appointment time. What would be the BEST action for the social worker to take? Negotiate a compromise to avoid escalation of the client's behavior Note the pattern in the client's file and continue to monitor the behavior Terminate the relationship and refer the client elsewhere Discuss the client’s perspective of the schedule Correct Answer Correct Answer This is an unscored item. The behavior of the client is contrary to agency policies, which have been explained to the client. To ignore this behavior or modify agency policy would encourage the client to continue challenging the rules. The behavior is not cause for termination of services. The appropriate response is (key D). 0 / 1 pts Question 111 Unanswered Unanswered The concept of "process" in social work refers to: the role of the social worker in dealing with the client recording what the social worker says to the client as they relate to each other the interactions of the client and social worker Correct Answer Correct Answer the sequence of case activities This is a straight recall question. In social work, process focuses on the dynamic between the client and social worker (key C). Options (A) and (B) are one-sided communication, and option (D) is a non-social work definition. 0 / 1 pts Question 112 Unanswered Unanswered What is the major benefit of intensive family preservation services? The opportunity for crisis intervention
Your preview ends here
Eager to read complete document? Join bartleby learn and gain access to the full version
  • Access to all documents
  • Unlimited textbook solutions
  • 24/7 expert homework help
11/3/23, 10:36 PM MSW Practice Test: MSW Advanced Standing-2022 Fall Quarter 08/29-11/20 https://waldenu.instructure.com/courses/17315/quizzes/13999 44/65 Empowerment of the family to participate in the process Correct Answer Correct Answer Long-term social work supervision with the family Removal of the primary focus from the family scapegoat Of the options available, empowerment of the family is the major benefit. The primary emphasis of this approach is not to alter the position of the scapegoat, nor is it to provide long-term supervision of the family. Crisis intervention may initially occur, but services offered provide longer term work with the family. 0 / 1 pts Question 113 Unanswered Unanswered Family members of a client who is elderly have concerns about recent changes in the client. These include memory loss for recent events, increasing irritability, confusion, and impaired speech. The client should be evaluated for: chemical dependency depressive episode neurocognitive disorder Correct Answer Correct Answer dietary habits Although options (A), (B), and (D) could possibly contribute to some of the client’s symptoms, it is MOST likely that the symptoms are caused by a neurocognitive disorder. A neurocognitive problem should be ruled out prior to considering other possibilities. 0 / 1 pts Question 114 Unanswered Unanswered During a supervised visit between a parent and small child, the parent suddenly becomes angry and verbally threatens the child and the social worker. The social worker should FIRST: encourage the parent to ventilate the anger attempt to defuse the situation by talking calmly to the parent secure the safety of the child by removing the child from the room Correct Answer Correct Answer terminate the visit and reschedule it for another time Options (A), (B), and (D) could potentially increase the parent’s anger. The social worker’s priority is to ensure the safety of the child by removing him or her from the room. 0 / 1 pts Question 115 Unanswered Unanswered
Your preview ends here
Eager to read complete document? Join bartleby learn and gain access to the full version
  • Access to all documents
  • Unlimited textbook solutions
  • 24/7 expert homework help
11/3/23, 10:36 PM MSW Practice Test: MSW Advanced Standing-2022 Fall Quarter 08/29-11/20 https://waldenu.instructure.com/courses/17315/quizzes/13999 45/65 What is the PRIMARY reason that maintenance of client records is considered to be a critical part of social work practice? It provides access to information to meet governmental requirements. It establishes accountability that the services were actually provided. Correct Answer Correct Answer It allows the social worker to debrief after difficult sessions. It facilitates transfer of records to other agencies. The client record provides documentation of services offered to the client. It may be useful in situations described in options (A), (C), and (D), but these do not reflect the PRIMARY purpose of record keeping. 0 / 1 pts Question 116 Unanswered Unanswered On an initial home health visit, a social worker learns that the client has diabetes and is in need of specialized foot care and nail trimming. What health professional can BEST provide these services? Osteopath Physical therapist Podiatrist Correct Answer Correct Answer Nurse This is a recall question that requires knowledge of multidisciplinary skills and scopes of practice, information that social workers should have. A podiatrist (key C) specializes in foot care. 0 / 1 pts Question 117 Unanswered Unanswered In an interview with a social worker, a client exhibits depressed affect, confusion about what is wrong, and ambivalence about requesting help. The social worker should FIRST: attempt to establish trust Correct Answer Correct Answer refer for a medical evaluation obtain a social history consult with the supervisor
Your preview ends here
Eager to read complete document? Join bartleby learn and gain access to the full version
  • Access to all documents
  • Unlimited textbook solutions
  • 24/7 expert homework help
11/3/23, 10:36 PM MSW Practice Test: MSW Advanced Standing-2022 Fall Quarter 08/29-11/20 https://waldenu.instructure.com/courses/17315/quizzes/13999 46/65 Because the client is unsure about coming for assistance, (key A) would be the most effective response. This must be done prior to obtaining a social history or referring the client for a medical exam. It is premature to consult with the supervisor. 0 / 1 pts Question 118 Unanswered Unanswered Which social work skill BEST communicates acceptance to a client? Demonstrating concern Using normalizing statements Using nonjudgmental statements Correct Answer Correct Answer Conveying competence Use of nonjudgmental statements (key C) conveys acceptance to the client by encouraging the client to speak openly without concern about the social worker’s opinion. While options (A) and (B) are both generally supportive, they do not communicate acceptance of the client and his or her unique situation. Acceptance does not involve option (D), which is a different issue. 0 / 1 pts Question 119 Unanswered Unanswered A distraught client walks into a mental health agency asking for help. The client tells the social worker about going to several other mental health agencies, saying they did not help. The social worker should FIRST: make an appointment for the client to receive a psychological assessment offer to talk with the client about previous treatment experiences Correct Answer Correct Answer encourage the client to return to one of the other agencies ask the client to sign releases of information for the other agencies The social worker should begin by starting where the client is and talking about the previous experiences that the client presents as part of the problem (key B). Options (A) and (D) might be done later, and option (C) is not indicated as something that should be done at this point. 0 / 1 pts Question 120 Unanswered Unanswered A high school social worker meets with a student whose parent became unemployed the previous month. The student is upset because the parent has been struggling to provide the family with adequate meals since then. After validating the student's feelings, what should the social worker do NEXT?
Your preview ends here
Eager to read complete document? Join bartleby learn and gain access to the full version
  • Access to all documents
  • Unlimited textbook solutions
  • 24/7 expert homework help
11/3/23, 10:36 PM MSW Practice Test: MSW Advanced Standing-2022 Fall Quarter 08/29-11/20 https://waldenu.instructure.com/courses/17315/quizzes/13999 47/65 Report the student's parent to legal authorities Explore the possibility of the student obtaining a part-time job Investigate the cause of the parent's unemployment Identify community resources to assist the family Correct Answer Correct Answer Because the family is struggling with a basic need, food, (key D) is correct. According to the hierarchy of needs, those basic physical necessities come before emotional needs. Options (A) and (C) are also actions that could antagonize the family and may not be appropriate even when (key D) has been done. 0 / 1 pts Question 121 Unanswered Unanswered A social worker's 86 -year-old client is partially disabled by a stroke. To be able to live at home, the client needs to relearn personal hygiene and feeding skills. What professional can BEST help the client with these needs? Cognitive therapist Occupational therapist Correct Answer Correct Answer Behavioral therapist Physical therapist Cognitive therapists work with mental functioning. Behavioral therapists work with changing specific behaviors, and physical therapists work with recovery from physical injury and movement. The only appropriate option is (key B). Occupational therapists assist clients in relearning daily functioning skills such as feeding and personal hygiene. 0 / 1 pts Question 122 Unanswered Unanswered A client in a domestic violence shelter wants to return to an abusive partner. The client is concerned that the social worker will not approve. What can the social worker do to BEST demonstrate acceptance of the client's choice? Refer the client to other services Encourage the client to discuss the decision Correct Answer Correct Answer Outline concerns about the risks of returning to the partner Allow the client to direct the conversation (Key B) is required by the social work value of self-determination. It also allows the client to further explore the decision, since support of the decision is different from acceptance.
Your preview ends here
Eager to read complete document? Join bartleby learn and gain access to the full version
  • Access to all documents
  • Unlimited textbook solutions
  • 24/7 expert homework help
11/3/23, 10:36 PM MSW Practice Test: MSW Advanced Standing-2022 Fall Quarter 08/29-11/20 https://waldenu.instructure.com/courses/17315/quizzes/13999 48/65 0 / 1 pts Question 123 Unanswered Unanswered A psychologist contacts a social worker to ask the worker to fax the social history of a former client to assist with an evaluation. The social worker should: obtain the client’s permission to share the information Correct Answer Correct Answer use an old release signed by the client to share the information provide the information verbally to the psychologist refuse to fax the information for reasons of confidentiality Even though the information requested pertains to a former client, the social worker must have the client’s permission before releasing the information. 0 / 1 pts Question 124 Unanswered Unanswered A social worker is obtaining a social history on a recently admitted patient. The social worker should FIRST address: family history vocational status the presenting problem Correct Answer Correct Answer any current medications Initial discussion of the client’s presenting problem will provide direction for the information to be included in the social history. 0 / 1 pts Question 125 Unanswered Unanswered A Tongan-speaking client was scheduled for a court ordered evaluation. She failed to attend the appointment even though transportation was provided. The driver reported nobody was home. The client said she was in the house asleep. The social worker should FIRST assess for: substance abuse motivation depression communication Correct Answer Correct Answer
Your preview ends here
Eager to read complete document? Join bartleby learn and gain access to the full version
  • Access to all documents
  • Unlimited textbook solutions
  • 24/7 expert homework help
11/3/23, 10:36 PM MSW Practice Test: MSW Advanced Standing-2022 Fall Quarter 08/29-11/20 https://waldenu.instructure.com/courses/17315/quizzes/13999 49/65 The social worker must FIRST determine whether or not the client understood the expectations. Where the client’s first language differs from that of the dominant culture, dealing first with communication issues is paramount. 0 / 1 pts Question 126 Unanswered Unanswered Social workers anticipating the interruption of services to clients who are still in need should: withdraw services gradually counsel with them in private evaluate unmet goals assist in making arrangements for continuation of services Correct Answer Correct Answer Social workers are obligated to assist clients through referrals when there is an ongoing need that will no longer be met. The code of ethics requires that (key D) be done. 0 / 1 pts Question 127 Unanswered Unanswered A child living in a family with an alcoholic member is MOST likely to experience: sexual abuse attention-deficit/hyperactivity disorder depression Correct Answer Correct Answer poverty While (A), (B), and (D) may occur in a family with an alcoholic member, the child is MOST likely to experience depression (key C). 0 / 1 pts Question 128 Unanswered Unanswered When an incident of parental abuse of a child occurs, the social worker should FIRST determine the: extent of alcohol/substance abuse in the family amount of bruising, discoloration, or fractures risk of future harm Correct Answer Correct Answer
Your preview ends here
Eager to read complete document? Join bartleby learn and gain access to the full version
  • Access to all documents
  • Unlimited textbook solutions
  • 24/7 expert homework help
11/3/23, 10:36 PM MSW Practice Test: MSW Advanced Standing-2022 Fall Quarter 08/29-11/20 https://waldenu.instructure.com/courses/17315/quizzes/13999 50/65 psychosocial history of both parents It is necessary to FIRST determine whether or not the child is in immediate danger (key C). Assessment of the child’s injuries should be completed by medical personnel. Options (A) and (D) would be part of a subsequent investigation, and family assessment and would not be the FIRST step for the social worker to take. 0 / 1 pts Question 129 Unanswered Unanswered According to attachment theory, what is the PRIMARY basis of a person's internal model of relationship patterns? Present circumstances Ongoing positive appraisals Early childhood experiences Correct Answer Correct Answer Perceptions of others' behaviors (Key C) is correct, according to attachment theory. A person’s relationships are determined by his or her attachment to a caregiver in early childhood. The other options, (A), (B), and (C), may be involved in the formation of relationship patterns but are not PRIMARY. 0 / 1 pts Question 130 Unanswered Unanswered A social worker meets with a client for the first time. The client is a member of a minority group. Although the client's issues center around depression, the social worker suspects that some of the concerns are related to racial discrimination. What is the MOST appropriate way for the social worker to explore this impression? Directly ask the client about personal exposure to racist behaviors Question the client about personal feelings about racial group membership Concentrate on building a relationship within which the client can voice racial concerns Correct Answer Correct Answer Assume that the client will discuss the issue if it relates to the depression This is an unscored item. Before directly questioning the client about racially related themes, the social worker must begin by establishing rapport with the client (key C). Option (D) is incorrect because it presents the social worker acting on assumptions, which is inappropriate. 0 / 1 pts Question 131 Unanswered Unanswered In a longitudinal research design, a panel study would involve:
Your preview ends here
Eager to read complete document? Join bartleby learn and gain access to the full version
  • Access to all documents
  • Unlimited textbook solutions
  • 24/7 expert homework help
11/3/23, 10:36 PM MSW Practice Test: MSW Advanced Standing-2022 Fall Quarter 08/29-11/20 https://waldenu.instructure.com/courses/17315/quizzes/13999 51/65 double blind methodology the use of a control variable in the study examination of the same subjects over time Correct Answer Correct Answer comparison of change in sample subgroups over time This is a recall question that requires knowledge of the definition of a longitudinal design. Longitudinal studies examine participants over a period of time (key C). 0 / 1 pts Question 132 Unanswered Unanswered Which of the following behaviors of an eight-month-old baby might indicate a delay in normal physical growth and development? Child is feeding self a cracker. Child does not speak a word. Child drops a toy and looks to find it. Child does not sit up without assistance. Correct Answer Correct Answer Options (A), (B), and (C) are normal behaviors for a child of eight months. Of the options, (key D) is the only one that indicates possible delayed development. A child eight months of age should be able to sit unassisted. 0 / 1 pts Question 133 Unanswered Unanswered A social worker meets with a patient who has cancer and is being cared for by a relative. The relative, who makes minimum wage, provides the sole financial support for the household. The patient complains that the relative does not provide enough for the patient financially. What action would be the MOST appropriate for the social worker to take? Explain to the patient that options are limited Determine if there is another caretaker available Assist the patient with accessing community resources Correct Answer Correct Answer Refer the patient to a support group Exploration of community resources would be the first step to take with this client. There is not enough information in the stem to support any of the other options as correct answers.
Your preview ends here
Eager to read complete document? Join bartleby learn and gain access to the full version
  • Access to all documents
  • Unlimited textbook solutions
  • 24/7 expert homework help
11/3/23, 10:36 PM MSW Practice Test: MSW Advanced Standing-2022 Fall Quarter 08/29-11/20 https://waldenu.instructure.com/courses/17315/quizzes/13999 52/65 0 / 1 pts Question 134 Unanswered Unanswered The type of hallucination MOST prevalent for individuals diagnosed with schizophrenia is: auditory Correct Answer Correct Answer tactile visual olfactory While it is possible for a person with schizophrenia to have any of these, auditory hallucinations (key A) are MOST common. 0 / 1 pts Question 135 Unanswered Unanswered A social worker works with a large number of gay and lesbian clients. The social worker notices that many of them have a common issue of low self-esteem stemming from the perception that society labels them as "bad." How can the social worker BEST address this situation? Lobby for new laws to fight against homophobia Educate community members on gay and lesbian issues Refer the clients to a gay or lesbian therapist Offer a gay and lesbian support group Correct Answer Correct Answer The offer of a support group (key D) is correct because it would create an immediate impact to provide support for an often marginalized population. Options (A) and (B) are opportunities to advocate by working on a macro level, but any benefits are very long-term. Option (C) would be avoiding the social worker’s responsibility and would be inappropriate. 0 / 1 pts Question 136 Unanswered Unanswered An agency's board of directors meets with the social work director to discuss its recommendation for a management position. The team is reluctant to recommend the most qualified applicant due to concerns that the applicant's deafness will prevent effective performance in the position. What should the social work director do? Support the management team's position by offering the position to the second strongest applicant Explore necessary reasonable accommodations when offering the position to the applicant Correct Answer Correct Answer Schedule a second interview with the applicant to further assess skills and ability to perform in the position Offer the position to the applicant on a probationary basis pending physician's approval
Your preview ends here
Eager to read complete document? Join bartleby learn and gain access to the full version
  • Access to all documents
  • Unlimited textbook solutions
  • 24/7 expert homework help
11/3/23, 10:36 PM MSW Practice Test: MSW Advanced Standing-2022 Fall Quarter 08/29-11/20 https://waldenu.instructure.com/courses/17315/quizzes/13999 53/65 This is an unscored item. A physical disability is not a barrier to employment unless it prevents the person from completing requirements of the job. Making reasonable accommodations for members of the workforce who have a disability is a legal requirement. 0 / 1 pts Question 137 Unanswered Unanswered Using a strengths perspective, the FIRST step in the assessment of a client's ability to benefit from intervention is to determine the client's: history of problem-solving cognitive level of functioning emotional response to the problems definition of the problem situation Correct Answer Correct Answer The social worker cannot be of assistance without FIRST allowing the client to define the problem and possible ways to solve it (key D). Options (A), (B), and (C) may become evident during the definition of the problem situation (key D). They are not the real FIRST step. A therapeutic relationship should always start with defining the problem. 0 / 1 pts Question 138 Unanswered Unanswered A ten -year-old child is violent toward other children at school and intimidates them into giving away their lunches. The BEST course of action for the school social worker is to: obtain a report from the teacher about the child's behavior request a psychosocial assessment of the child request a conference with the parents and teacher Correct Answer Correct Answer meet with class to teach conflict resolution skills The social worker must first evaluate the child’s present circumstances at home and at school, and begin to involve the parents in the problem-solving process. A psychosocial assessment of the child would follow. Option (D) does not address the issues specifically related to the child. 0 / 1 pts Question 139 Unanswered Unanswered During the latter stages of an individual's alcohol addiction, what are immediate family members MOST likely to do? Demonstrate high-level reality testing abilities
Your preview ends here
Eager to read complete document? Join bartleby learn and gain access to the full version
  • Access to all documents
  • Unlimited textbook solutions
  • 24/7 expert homework help
11/3/23, 10:36 PM MSW Practice Test: MSW Advanced Standing-2022 Fall Quarter 08/29-11/20 https://waldenu.instructure.com/courses/17315/quizzes/13999 54/65 Disengage from the addicted person Blame the addicted person for the alcoholic behavior Assume many of the addicted person's responsibilities Correct Answer Correct Answer According to the stages of addiction, the latter stage typically involves the inability of an individual to fulfill responsibilities, so family members must assume them (key D). The other three options represent earlier stages of addiction. 0 / 1 pts Question 140 Unanswered Unanswered In an effort to make more effective referrals for clients, a social worker in a community service center spends several days visiting local agencies and meeting service providers. This practice is an example of: consultation networking Correct Answer Correct Answer collaboration evaluation Social work networking (key B) involves meeting other providers in order to broaden collaborative efforts for the benefit of clients. Option (A) involves obtaining information from a colleague or colleagues who may be more familiar with a particular issue. Collaboration, option (C), is people working together on an already identified project. 0 / 1 pts Question 141 Unanswered Unanswered The adult children of a person recently diagnosed with Alzheimer's disease will MOST likely demonstrate which of the following emotional reactions? Immortality versus extinction Isolation Depression Denial Correct Answer Correct Answer Individuals experience the stages of grief in learning of the expected loss of the person they have known, as may occur with Alzheimer’s, in much the way they would a loss by death. Denial (key D) is the common initial reaction in these stages. Options (B) and (C) may develop later in the grief process. Option (A) is a normal stage of development and is not applicable here.
Your preview ends here
Eager to read complete document? Join bartleby learn and gain access to the full version
  • Access to all documents
  • Unlimited textbook solutions
  • 24/7 expert homework help
11/3/23, 10:36 PM MSW Practice Test: MSW Advanced Standing-2022 Fall Quarter 08/29-11/20 https://waldenu.instructure.com/courses/17315/quizzes/13999 55/65 0 / 1 pts Question 142 Unanswered Unanswered Which of the following factors is MOST important in determining family adjustment to a child’s disability? Geographical proximity of the family doctor The availability of social work services The onset of the child's condition Correct Answer Correct Answer Accessibility of the family dwelling This is an unscored item. Being geographically close to the doctor (A) might make it easier for the infant to receive medical care, but it would not determine the family’s ability to adjust to the disability. This is the same for having available social work services (B). The accessibility of the family’s dwelling (D) is not a determining factor in how the family will adjust and would not necessarily be a concern for the family. The sudden or gradual onset of the condition (key C) is the best factor in indicating how the family will adjust. 0 / 1 pts Question 143 Unanswered Unanswered A social worker meets with a client for an initial interview at a substance abuse center. The client reports a lengthy history of alcohol use and is now experiencing a reduced effect after consuming the same amount of alcohol. What aspect of alcohol use is the client describing? Addiction Tolerance Correct Answer Correct Answer Intoxication Abuse Tolerance (key B) is the body’s requirement for larger amounts of alcohol in order to achieve the same effect, or high. This question requires recall of the definition of tolerance. 0 / 1 pts Question 144 Unanswered Unanswered Which statement BEST describes the concept of privileged communication for social workers? It provides the legal grounds for confidentiality. Correct Answer Correct Answer It permits the social worker the discretion to share client information. It prevents client information from being shared at agency case conferences. It protects clients who report planning violent acts.
Your preview ends here
Eager to read complete document? Join bartleby learn and gain access to the full version
  • Access to all documents
  • Unlimited textbook solutions
  • 24/7 expert homework help
11/3/23, 10:36 PM MSW Practice Test: MSW Advanced Standing-2022 Fall Quarter 08/29-11/20 https://waldenu.instructure.com/courses/17315/quizzes/13999 56/65 This question requires recall of the definition of privileged communication. Privileged communication is the foundation of confidentiality and ensures clients that communication will be kept private except in cases of potential harm (key A). Option (C) is incorrect because there are legal provisions allowing necessary intra- agency communication. 0 / 1 pts Question 145 Unanswered Unanswered A client describes being very close to siblings when they were all young children but drifting apart emotionally in midlife. Since retiring, the client wishes to become closer to two siblings who reside in another town. The client is describing: an unhealthy desire to reconstruct family of origin emotions a need for closeness that would best be filled locally unrealistic expectations because of distance and time barriers a typical life course pattern at this stage of development Correct Answer Correct Answer During later adult years, it is not unusual for a person to reestablish contact with family of origin members. It is not a pathological desire but rather a way of establishing a sense of continuity and connectedness. (Key D) is correct. 0 / 1 pts Question 146 Unanswered Unanswered A social worker working with a troubled fifth grader and the family learns that the student has been suspended from school for two weeks. The social worker contacts the principal of the school to encourage an alternative punishment that would not further harm the student's grades. In this situation, the social worker is fulfilling the role of: mediator enabler advocate Correct Answer Correct Answer liaison The act of contacting the principal is advocacy on the part of the social worker. The social worker is attempting to effect a change that would benefit the client. 0 / 1 pts Question 147 Unanswered Unanswered A social worker is assisting a couple on a child protective services matter. The husband calls the social worker to tell her that he recently learned he is HIV positive. He asks the social worker to keep the information confidential. The social worker should FIRST:
Your preview ends here
Eager to read complete document? Join bartleby learn and gain access to the full version
  • Access to all documents
  • Unlimited textbook solutions
  • 24/7 expert homework help
11/3/23, 10:36 PM MSW Practice Test: MSW Advanced Standing-2022 Fall Quarter 08/29-11/20 https://waldenu.instructure.com/courses/17315/quizzes/13999 57/65 tell the wife the husband is HIV positive report the husband's status to the local health department assure the husband that everything he tells the social worker is confidential encourage the husband to inform his wife of his HIV status Correct Answer Correct Answer Although jurisdictional laws and regulations may govern what the social worker does with the information, the immediate FIRST action is to encourage the client to share the information with his wife. 0 / 1 pts Question 148 Unanswered Unanswered A social worker at a mental health agency is working with a person who is homeless. The social worker is concerned about the physical safety and health of the client. What action should the social worker take? Insist that the client seek admission to a shelter Refuse to provide services unless the client enters a shelter Describe available health care and housing options to the client Correct Answer Correct Answer Seek involuntary psychiatric commitment for the client This is an unscored item. Options (A), (B), and (D) provide the client with no choice. Providing housing and health care options from which to choose (key C) is a way to empower the client and help restore the worth and dignity of the individual. 0 / 1 pts Question 149 Unanswered Unanswered During a meeting with a social worker, the client describes an inability to sleep at night and a decreased appetite. In addition to assessing the client's symptoms, the social worker should also FIRST assess for: cultural values environmental stressors Correct Answer Correct Answer the client's self-concept the social worker's role with the client An initial social work assessment always includes exploration of environmental stressors. Exploration of the client’s cultural values and self-concept will occur later. Although the client is given an explanation of the social worker’s role early in the contact, this does not constitute an assessment of the relationship.
Your preview ends here
Eager to read complete document? Join bartleby learn and gain access to the full version
  • Access to all documents
  • Unlimited textbook solutions
  • 24/7 expert homework help
11/3/23, 10:36 PM MSW Practice Test: MSW Advanced Standing-2022 Fall Quarter 08/29-11/20 https://waldenu.instructure.com/courses/17315/quizzes/13999 58/65 0 / 1 pts Question 150 Unanswered Unanswered A social work manager is preparing to work on the program's budget for the coming year. There is a better chance of it being funded if the social worker develops a budget that requests: precise funds needed for different categories of expenditures Correct Answer Correct Answer an increase for more than is needed to secure adequate funding funds for overall expenditures without showing each budgeted item separately the same funding as was approved in the previous year's budget This is an unscored item. (Key A) is the only choice that provides a realistic portrayal of the agency’s budgetary needs for the coming year. Options (B) and (C) are not good financial practices. Option (D) does not allow for necessary increases in operating expenses caused by inflation. 0 / 1 pts Question 151 Unanswered Unanswered Piaget's period of "formal operations," during which an individual becomes capable of thinking conceptually and using hypotheses, begins during approximately which developmental time frame? Early adolescence Correct Answer Correct Answer Latency Early adulthood Late adolescence This is a recall question. According to Piaget, the period of formal operations begins to occur in early adolescence (key A). 0 / 1 pts Question 152 Unanswered Unanswered Knowledge of family life cycle theory enables social workers to: predict the psychosocial development of individual family members determine levels of family developmental task accomplishment Correct Answer Correct Answer assess dysfunctional family interactions explain family violence risks
Your preview ends here
Eager to read complete document? Join bartleby learn and gain access to the full version
  • Access to all documents
  • Unlimited textbook solutions
  • 24/7 expert homework help
11/3/23, 10:36 PM MSW Practice Test: MSW Advanced Standing-2022 Fall Quarter 08/29-11/20 https://waldenu.instructure.com/courses/17315/quizzes/13999 59/65 Family life cycle theory looks at the progression of the family from stage to stage as it achieves developmental tasks (key B). It does not include the other options. 0 / 1 pts Question 153 Unanswered Unanswered An intake social worker at a family service agency realizes that many people who make initial contact with the agency are unable to come for service because the agency is not open during evening hours. Agency policy prohibits evening operation. In attempting to change this policy, what should the social worker do FIRST? Request that the concern be placed on the agenda for the next staff meeting Correct Answer Correct Answer Start a petition to extend the agency’s hours Explore grants for funding evening hours Suggest a cost-benefit analysis This is an unscored item. The FIRST step to take is to discuss the concern with the entire staff. This will provide a forum for discussion of the issue as well as a sharing of possible ways to address the concern. The remaining options may become steps to take at a later date. 0 / 1 pts Question 154 Unanswered Unanswered The MOST effective way to safeguard against differences between experimental and control groups is to: match subjects in the different treatment conditions partial out differences using analysis of covariance randomly assign subjects to the treatment conditions Correct Answer Correct Answer subject the data to factor analysis (Key C) assures there is no bias in the group assignment of subjects, thus increasing the chance of similarity of the two groups. 0 / 1 pts Question 155 Unanswered Unanswered The child in an alcoholic family who is funny and consistently entertains the family members is assuming the role of the: scapegoat hero
Your preview ends here
Eager to read complete document? Join bartleby learn and gain access to the full version
  • Access to all documents
  • Unlimited textbook solutions
  • 24/7 expert homework help
11/3/23, 10:36 PM MSW Practice Test: MSW Advanced Standing-2022 Fall Quarter 08/29-11/20 https://waldenu.instructure.com/courses/17315/quizzes/13999 60/65 lost child mascot Correct Answer Correct Answer Family members in an alcoholic family tend to take on roles. In this case, the child’s role is that of mascot (key D), distracting from family dysfunction by entertaining. Option (A), the scapegoat, becomes the recipient of blame, and (B), the hero, reacts by overachieving. In option (C), the lost child simply becomes invisible. 0 / 1 pts Question 156 Unanswered Unanswered A father meets with a social worker to talk about problems he is having with his teenager. At different points during the meeting, the social worker restates the issues in more concise and simplified terms. The social worker is using what practice technique? Reflecting Feedback Paraphrasing Correct Answer Correct Answer Probing Paraphrasing (key C) expresses the relevant points of what the client has said and provides validation that the client was heard. Reflecting involves stating what the client is feeling at that moment. Feedback is an objective analysis of actions and their resulting effects. The social worker is not using probing, which would involve further exploration of the client’s feelings, experience or situation. 0 / 1 pts Question 157 Unanswered Unanswered Group supervision is MOST effective when it: increases the comfort level of staff in their surroundings orients less experienced staff to role expectations enables staff to share failures with peers to improve practice is able to address common learning needs of supervisees Correct Answer Correct Answer The goal of group supervision is not to increase staff comfort. While options (B) and (C) may exist during the group supervisory sessions, this type of supervision is MOST effective when those being supervised have their common learning needs addressed. 0 / 1 pts Question 158 Unanswered Unanswered
Your preview ends here
Eager to read complete document? Join bartleby learn and gain access to the full version
  • Access to all documents
  • Unlimited textbook solutions
  • 24/7 expert homework help
11/3/23, 10:36 PM MSW Practice Test: MSW Advanced Standing-2022 Fall Quarter 08/29-11/20 https://waldenu.instructure.com/courses/17315/quizzes/13999 61/65 To prepare a preschooler for hospitalization, the social worker should FIRST: emphasize the pleasant aspects of the experience arrange for the child to take a preliminary trip to the hospital Correct Answer Correct Answer ask physicians to explain to the child what will happen in the hospital suggest that parents remain calm (Key B) will help prepare the child emotionally for the hospitalization. It would be unfair to the child to minimize the negative aspects of hospitalization and emphasize the positive aspects (A), as this may not reflect the reality the child will experience. Option (C) is not likely to be helpful to a child with little or no basis for understanding medical procedures. There is no mention of the parents being distraught, so option (D) would be incorrect. 0 / 1 pts Question 159 Unanswered Unanswered What characterizes the stage of immobility in a crisis situation? An increase in resource mobilization The absence of emotional stability and control in daily functioning An inability to adapt to changes or cope with current stressors Correct Answer Correct Answer A state of homeostasis in the client system A person in the initial stages of a crisis is likely to experience some degree of immobility because previously used coping behaviors are not effective. The immobility interferes with the identification and use of resources. A period of homeostasis does not occur until the crisis has been successfully resolved 0 / 1 pts Question 160 Unanswered Unanswered In comparison with that of an adult, the social history of a child MOST likely contains more extensive information on which of the following categories? Identifying information on the client History of the presenting problem Observations by a social worker Developmental history of the client Correct Answer Correct Answer
Your preview ends here
Eager to read complete document? Join bartleby learn and gain access to the full version
  • Access to all documents
  • Unlimited textbook solutions
  • 24/7 expert homework help
11/3/23, 10:36 PM MSW Practice Test: MSW Advanced Standing-2022 Fall Quarter 08/29-11/20 https://waldenu.instructure.com/courses/17315/quizzes/13999 62/65 0 / 1 pts Question 161 Unanswered Unanswered In social work practice, color blindness means disregarding: socioeconomic differences cultural commonalties individual personality characteristics racial and ethnic differences Correct Answer Correct Answer The correct answer (key D) is the only option that deals with color or racial differences. Color blindness assumes that visible outward appearances, such as race and ethnicity, are not indicators of individual differences. Option (A) considers class rather than race, and neither class, cultural commonalties (B), nor individual personality traits (C) are visible, outward differences. 0 / 1 pts Question 162 Unanswered Unanswered Siblings are referred to the school social worker. Teachers are concerned that the children are often absent, have hygiene problems, and have not been participating in school activities. The social worker should FIRST: make a referral to the housing authority ask the children about their home environment Correct Answer Correct Answer develop a contract with the parents, outlining expectations interview the children privately to investigate for possible abuse Information in the stem indicates that the children are possibly experiencing parental neglect. To make this determination, the social worker would FIRST meet with the children. Options (A) and (C) are clearly premature actions to take. Absence from school and poor hygiene are insufficient indicators of abuse, so option (D) is not correct. 0 / 1 pts Question 163 Unanswered Unanswered Which of the following documents is a means used by social workers to facilitate implementation of an intervention plan with a client? A service outcome checklist A protocol statement A written service contract Correct Answer Correct Answer A termination agreement
Your preview ends here
Eager to read complete document? Join bartleby learn and gain access to the full version
  • Access to all documents
  • Unlimited textbook solutions
  • 24/7 expert homework help
11/3/23, 10:36 PM MSW Practice Test: MSW Advanced Standing-2022 Fall Quarter 08/29-11/20 https://waldenu.instructure.com/courses/17315/quizzes/13999 63/65 This is an unscored item. A written service contract (key C) implies an agreement between the client and the social worker on goals to be worked on and services to be provided. 0 / 1 pts Question 164 Unanswered Unanswered A social worker visits an elderly client whose grandchildren also live in the client's home. The client is unable to perform activities of daily living and is found lying in a soiled bed in a windowless room. When the social worker questions the grandchildren, it is discovered that no family members are home during the day. The social worker should: make a report immediately to adult protective services Correct Answer Correct Answer give the family a list of nursing homes in the area for consideration contact other family members in the area and inform them of the situation insist that at least one grandchild remain at home during the day Options (B), (C), and (D) are tasks the social worker may do at some point, but immediate reporting (key A) is a legal and ethical obligation. 0 / 1 pts Question 165 Unanswered Unanswered A 20 -year-old contacts a social worker who is the parent of a friend. The young adult discloses to the social worker a previous history of sexual abuse and social service involvement. The young adult asks to come to the social worker's home to seek help related to the abuse. In response, the social worker should: talk with the social service agency to review the records consult the previous social worker who worked with the young adult allow the young adult to visit as requested explain the conflict of roles and offer to arrange a referral Correct Answer Correct Answer It is a conflict of interest and an ethical issue for the social worker to consider any of the first three options. The only acceptable option, from an ethical perspective, is for the social worker to explain the conflict of roles and arrange a referral. 0 / 1 pts Question 166 Unanswered Unanswered A social worker notices bruises on a client's arms and asks about them. The client begins to cry and states, "My spouse didn't mean to hurt me." The social worker should FIRST determine:
Your preview ends here
Eager to read complete document? Join bartleby learn and gain access to the full version
  • Access to all documents
  • Unlimited textbook solutions
  • 24/7 expert homework help
11/3/23, 10:36 PM MSW Practice Test: MSW Advanced Standing-2022 Fall Quarter 08/29-11/20 https://waldenu.instructure.com/courses/17315/quizzes/13999 64/65 resources available to the client in the community the client's wishes concerning the violence the type and extent of abuse patterns in the family the potential of immediate danger to the client Correct Answer Correct Answer Physical safety and security (key D) are the FIRST concerns in the hierarchy of needs. The other options are potential tasks, but not the FIRST. 0 / 1 pts Question 167 Unanswered Unanswered A client who has recently moved from a rural area to a large metropolitan city has been referred to a social worker for high-risk pregnancy services. She has missed or has been late for all of her appointments. Based on limited contact, the social worker does not view the client as noncompliant. The social worker should FIRST: find an agency that is closer to the client's home explore the client's adjustment to living in the city Correct Answer Correct Answer ask the social work supervisor for guidance develop a contract addressing missed appointments The social worker should begin with exploring the client’s adjustment to recent changes (key B), allowing the client to express possible barriers and concerns. Understanding the client’s issues with keeping her appointments is necessary before determining the next steps, which might be any of the other options. 0 / 1 pts Question 168 Unanswered Unanswered A social worker is unable to understand a client who has limited English proficiency. What approach would be MOST helpful for the social worker to use? Explore with the client the use of an interpreter Correct Answer Correct Answer Ask the client to bring in a trusted family member to interpret Ask the client to speak more slowly and clearly Suggest a referral to another agency
Your preview ends here
Eager to read complete document? Join bartleby learn and gain access to the full version
  • Access to all documents
  • Unlimited textbook solutions
  • 24/7 expert homework help
11/3/23, 10:36 PM MSW Practice Test: MSW Advanced Standing-2022 Fall Quarter 08/29-11/20 https://waldenu.instructure.com/courses/17315/quizzes/13999 65/65 It would not be in the client’s best interest necessarily to have a family member interpret what the client is saying, as there may be confidentiality issues to consider. It is not the client’s responsibility to change his or her level of language ability to ensure that the social worker is able to understand. While a referral to another agency may be necessary at some point, it should take place only after exploring other options such as using an interpreter. 0 / 1 pts Question 169 Unanswered Unanswered The basic function of the mental status exam is to assess whether there is a need to refer clients for: substance abuse screening psychological intelligence testing forensic screening psychiatric or neurological examination Correct Answer Correct Answer This is a recall question. A mental status exam assesses the mental health and neurological functioning of the client, to determine whether there is a need for (key D). Option (A) is specific to drug abuse, (B) is an intelligence test, and (C) is specific to the court system and does not indicate whether a client is functioning with basic mental health. 0 / 1 pts Question 170 Unanswered Unanswered A social worker providing substance abuse services is concerned that an adolescent client may be actively using drugs. After discussing the concern with the client, the social worker remains unsure about the client's drug use. The social worker should NEXT interview the client’s: teachers peers parents Correct Answer Correct Answer grandparents This is an unscored item. The social worker should speak with the parents of the adolescent (key C) before obtaining information from resources outside the family. It would not be appropriate to question teachers, the grandparents, or the client’s peers about the possible drug use. Quiz Score: 21 out of 170
Your preview ends here
Eager to read complete document? Join bartleby learn and gain access to the full version
  • Access to all documents
  • Unlimited textbook solutions
  • 24/7 expert homework help